SlideShare a Scribd company logo
1 of 135
Download to read offline
GoutGout
Alexa Carlson, PharmD, BCPS
Northeastern University
1. Define and explain commonly used medical terminology and abbreviations within the
context of a patient case.
2. Describe and correlate signs and symptoms, epidemiology, risk factors, pathogenesis,
natural history of disease, clinical course, etiology, and prevention and treatment of
commonly encountered diseases.
3. Apply scientific knowledge and principles of medicinal chemistry, pharmacology,
pharmaceutics, pharmacogenomics and pharmacokinetics to the design of rational
therapeutic strategies
4. Identify evidence based, patient specific therapeutic goals and outcome measures for
a patient started on a drug therapy regimen.
5. Apply evidenced based medicine to compare and contrast therapeutic options for a
given medical problem and/or disease state in terms of safety, efficacy (including
clinical evidence), ease of administration, cost, and other patient specific factors.
6. Design and defend a patient-specific therapeutic regimen, which is safe, efficacious
and practical. For each drug selected, include drug name (generic and brand) dose,
route, frequency and time of administration, and duration of therapy when
applicable.
7. Predict the influence of selected drugs on a patient’s other diseases or problems and
outline which agents should be avoided or monitored more carefully.
Neogi T. Gout. N Engl J Med. 2011:364(5):443-452.
Khanna D, Fitzgerald JD, Khanna PP, et al. 2012 American College of Rheumatology
Guidelines for Management of Gout. Part 1: Systematic Nonpharmacologic and
Pharmacologic Therapeutic Approaches to Hyperuricemia. Arthritis Care & Research.
2012:64(10);1431-1446.
Khanna D, Khanna PP, Fitzgerald JD, et al. 2012 American College of Rheumatology
Guidelines for Management of Gout. Part 2: Therapy and Antiinflammatory
Prophylaxis of Acute Gouty Arthritis. Arthritis Care & Research. 2012:64(10);1447-
1461.
Jordan DM, Cameron JS, Snaith M, et al. British Society for Rheumatology Guideline
for the Management of Gout. Rheumatology. 2007;46:1372-1374.
Zhang W, Doherty M, Pascual E, et al. EULAR evidence based recommendations for
gout. Part I: Report of a task force of the standing committee for international clinical
studies including therapeutics (ESCISIT). Ann Rheum Dis. 2006l65:1301-1311.
Zhang W, Doherty M, Bardin T, et al. EULAR evidence based recommendations for
gout. Part II: Management. Report of a task force of the EULAR Standing Committee
for International Clinical Studies Including Therapeutics (ESCISIT). Ann Rheum Dis.
2006;65:1312-1324.
Roddy E, Zhang W, Doherty M. Concordance of the management of chronic gout in a
UK primary-care population with the EULAR gout recommendations. Ann Rheum Dis.
2007;66:1311-1315.
Gout is the most common cause of inflammatory
arthritis in the US
Substantial gaps in the management of gout
Significant lack of patient education and
adherence to management
Hyperuricemia: elevated serum uric acid
Tophus: a calculus containing sodium urate that
develops around fibrous tissue around joints, typically
in patients with gout
Podagra: a painful condition of the big toe caused by
gout
Uricase: An enzyme that oxidatively degrade uric acid,
thereby catalyzing conversion to soluble allantoin,
which is much more soluble than uric acid. This
enzyme is found in most animals, but not in humans.
Uricosuric medications: Medications administered to
increase the elimination of uric acid.
Elevated serum uric acid (SUA)
>7mg/dL (416μmol/L) at 37°C for men
>6mg/dL (357μmol/L) at 37°C for women
Considered to be the principle cause of gouty
arthritis
Not all patients with hyperuricemia develop
acute gout flares or chronic gouty complications
Clinical spectrum of disease:
Hyperuricemia
Recurrent acute arthritis
attacks due to monosodium
urate (MSU) crystals in synovial
fluid
Deposition of MSU into
articular and extra-articular
space (tophi)
Interstitial renal disease
Uric acid nephrolithiasis
Primarily caused by elevated
SUA levels, but can occur in
patients with normal SUA
levels.
http://www.healthinplainenglish.com/health
/musculoskeletal/gout/
http://www.assh.org/Public/HandConditions/Pag
es/GoutandPseudogout.aspx
“Disease of Kings”
Alexander the Great
King Henry VIII
Benjamin Franklin
Alexander Hamilton
Voltaire
Health.com List of 8 people with gout:
http://www.health.com/health/gallery/0,,2045
1892,00.html
Image from: antabusediaries.blogspot.com
Increasing prevalence of gout
Developed Countries > Developing Countries
3.9% of adults in the US (~8.3 million people)
$3.9 million in annual physician visits
Richette P, Bardin T. Gout. Lancet. 2010;375:318-28.
Date of download: 5/20/2014 Copyright © 2012 McGraw-Hill Medical. All rights reserved.
Purine metabolism. (HGPRT, hypoxanthine-guanine phosphoribosyltransferase; PRPP, phosphoribosyl pyrophosphate.)
Legend:
From: Chapter 74. Gout and Hyperuricemia
Pharmacotherapy: A Pathophysiologic
Approach, 9e, 2014
Uric acid is the end product of purine metabolism
Elevated uric acid can occur from over-production or
under-excretion of uric acid
Over-production:
Less common
Genetic abnormalities in enzymes related purine metabolism
Increased phosphoribosyl pyrophosphate synthetase (PRPP)
Decreased hypoxanthine-guanine phosphoribosylthransferase (HGPRT)
Increased cell turnover (cytotoxic chemotherapy, malignancies)
Under-excretion:
More common (~90%)
Decline in urinary excretion of uric acid
Can determine by measuring urine uric acid
>600mg/24h over-production
<600mg/24h under-excretion
Image from: www.goutrx.com
A. All patients with gout with progress to chronic
tophaceous gout
B. All patients with gout have hyperuricemia
C. Patients with gout will have recurrent acute
attacks separated by intercritical periods
D. Gout is primarily due to the overproduction of
uric acid
Increasing age
Sex (Male > Female)
Hyperuricemia
Obesity
High Blood Pressure
Injury
Fasting
Recent surgery
Foods/Drinks
Medications
Medical Conditions
Genetics
Foods high in purines
Foods and drinks with high fructose corn
syrup
Alcohol
Picture taken from: aldrodriguezliverfoundation.com
Diuretics (thiazide
and loop diuretics)
Cyclosporine and
tacrolimus
Low dose salicylates
(<2g/day)
Levodopa
Pyrazinamide
Cytotoxic
chemotherapy
Cyclosporine
Nicotinic Acid
(Niacin)
Ethambutol
Chronic renal insufficiency
Hypothyroidism
Renal Transplant
Hypertension
Coronary heart disease
Myeloproliferative disorders
Lymphoproliferative disorders
Lead toxicity
Starvation
Acromegaly
Down’s syndrome
Hypoxanthine-guanine
phosphoribosyltransferase deficiency
Diabetes mellitus
Volume depletion
Ketoacidosis
Lactic Acidosis
Hyper/Hypo-parathyroidism
Sarcoidosis
Preeclampsia
Starvation
Heart Failure
Acute alcoholism
Phosphoribosylpyrophosphate
synthetase overactivity
Pernicious anemia
Psoriasis
RT is a 48 year old male
PMH: HTN, Type II DM, CHF
SH: 5 pack-year history, drinks 2 beers per night
Current Medications: metoprolol XL 50mg po
daily, aspirin 81mg po daily, lisinopril 20mg po
daily, metformin 1000mg po BID, furosemide
40mg po daily
A. One
B. Two
C. Three
D. Four
Symptoms:Symptoms:
•Rapid onsetRapid onset
•Over night or after a trigger (alcohol,Over night or after a trigger (alcohol,
meats, diuretics)meats, diuretics)
•Excruciating painExcruciating pain
Signs:Signs:
•Monoarticular arthritis early on (mayMonoarticular arthritis early on (may
become polyarticular in late gout)become polyarticular in late gout)
•Often occurs in a joint in the lowerOften occurs in a joint in the lower
extremityextremity
•Redness/Swelling/Inflammation/WarmtRedness/Swelling/Inflammation/Warmt
h of the jointh of the joint
•FeverFever
•Elevated serum uric acid leukocytosisElevated serum uric acid leukocytosis
•Monosodium urate cyrstals in synovialMonosodium urate cyrstals in synovial
fluidfluid
http://knol.google.com/k/gout
• Differential Diagnosis:
– Pseudogout (calcium pyrophosphate crystals)
– Septic arthritis
– Rheumatoid arthritis
– Trauma
• Diagnosis:
– Evidence of monosodium urate (MSU) crystals by
joint aspiration is gold standard for definitive
diagnosis
– Often made on as a clinical diagnosis
Zhang W, Doherty M, Pascual E, et al. EULAR Evidence Based Recommendations for Gout. Part I: Diagnosis. Report of a Task Force of the Standing Committee for International Clinical
Studies Including Therapeutics (ESCISIT). Ann Rheum Dis. 2006;65:1301-11.
Gout cdm2 2014_class version(1)
• Level A: Supported by multiple (ie, more than
one) randomized clinical trials or meta-analyses
• Level B: Derived from a single randomized trial,
or nonrandomized studies
• Level C: Consensus opinion of experts, case
studies, or standard-of-care
Colchicine,Colchicine,
NSAIDs,NSAIDs,
CorticosteroidsCorticosteroids
http://rwglobal.com/~slpm/tipe/p
ictures/gout.jpg
• Acute flares are self-limiting
• Treatment objective is rapid relief of symptoms
• All therapy should be initiated within 24h of gout
symptom onset and continued for 1-2 weeks
• Do NOT discontinue urate lowering therapy in an
acute attack
Rheumatology. 2007;46:1372-1374. Ann Rheum Dis. 2006;65:1312-24. Arthritis Care Res. 2012:64(10);1431-1446.
Ann Rheum Dis. 2006;65:1312-24.
• Mechanism of Action: Peripheral inhibition of
COX leading to inhibition of prostaglandin
synthesis
• Dosing:
NSAID Typical Regimen
Etodolac 300mg Q12h
Fenoprofen 300-600mg Q6-8h
Ibuprofen 800mg Q6h
Indomethacin 25-50mg Q6h x 3d, then taper to Q12h x 4-7d
Ketoprofen 75mg Q6h
Naproxen 500mg Q12h x 3d, then 250-500mg daily x 4-7d
Piroxicam 20mg daily or divided Q12h
Sulindac 200mg Q12h x 7-10d
• Adverse Effects: increased BP, sodium and
water retention, gastritis, GI bleeding
• Contraindications/Precautions:
Ann Rheum Dis. 2006:65:;312-24. NEJM. 2011:364(5);443-452.
• FDA-Approved: indomethacin, naproxen,
sulindac
• NO NSAID has been shown to be superior to
another
• Role in Therapy:
– FIRST LINE for acute gouty attacks w/o
contraindication at high doses FIRST LINE alternative
option to colchicine for prophylaxis
• Mechanism of Action: anti-inflammatory
• Dosing:
Corticosteroid Typical Regimen
Prednisone 0.5 mg/kg/day x5-10d, then stop (Evidence A)
<OR>
0.5mg/kg/day x2-5d, then taper for 7-10d, then stop
(Evidence C)
Methylprednisolone Methylprednisolone oral dose pack (Evidence C)
Methylprednisolone IM 100-150mg daily x1-2d
Triamcinolone IM 60mg once, then oral prednisone (Evidence C)
Triamcinolone intra-articular 10-40mg (large joints) or 5-20mg (small joints) once
(Evidence B)
• Adverse Effects: hyperglycemia, leukocytosis,
fluid retention, impaired wound healing, GI
upset, insomnia
• Precautions: Infection, DM, peptic ulcer disease
• Role in Therapy:
– FIRST LINE therapy in acute gout
– Consider intra-articular corticosteroids in patients
with gout affecting 1-2 large joints
– SECOND LINE therapy in prophylaxis to NSAIDs and
colchicineJanssens HJEM, Janssen M, van de Lisdonk EH, et al. Use of Oral Prednisolone or Naproxen for the Treatment of Gout Arthritis: a
Double-Blind, Randomized Equivalence Trial. Lancet. 2008;371:1854-60.
Use of Oral Prednisolone or Naproxen for the Treatment of Gout Arthritis: a
Double-Blind, Randomized Equivalence Trial
Methods Randomized, double-blind, double-dummy, active-comparator,
controlled trial in monoarticular gout to assess equivalency
Intervention • Naproxen 500mg po BID x 5 days (n=59)
• Prednisolone 35mg po daily x 5 days (n=59)
Primary
Outcomes
Pain in the affected joint measured by the 100mm visual analogue scale
Results Reduction in mean pain score on visual analogue scale on day 4:
•44.7mm prednisolone
•46mm naproxen
•Difference 1.3mm (95% CI -9.8-7.1)
Relief of symptoms on day 4:
•Clinically significant improvements: 80% prednisolone and 87%
naproxen patients
•Complete relief of symptoms: 22% prednisolone and 17% naproxen
patients
Janssens HJEM, Janssen M, van de Lisdonk EH, et al. Use of Oral Prednisolone or Naproxen for the Treatment of Gout Arthritis: a
Double-Blind, Randomized Equivalence Trial. Lancet. 2008;371:1854-60.
Janssens HJEM, Janssen M, van de Lisdonk EH, et al. Use of Oral Prednisolone or Naproxen for the Treatment of Gout Arthritis: a Double-Blind, Randomized
Equivalence Trial. Lancet. 2008;371:1854-60.
Janssens HJEM, Janssen M, van de Lisdonk EH, et al. Use of Oral Prednisolone or Naproxen for the Treatment of Gout Arthritis: a
Double-Blind, Randomized Equivalence Trial. Lancet. 2008;371:1854-60.
• Mechanism of Action:
– May interfere with the intracellular assembly of the
inflammasome complex present in neutrophils and
monocytes that mediates the activation of
interlukin-1β
– Disruption of cytoskeletal functions through
inhibition of β-tubulin polymerization into
microtubules which prevents the activation,
degranulation, and migration of neutrophils
Colcrys [package insert]. Philadelphia, PA: AR Scientific, Inc; 2012.
Methods Multicenter, randomized, double-blind, placebo-
controlled, parallel-group study
Treatment Arms
(n=184)
Placebo (n=59)
Low-dose colchicine (1.2 mg po with 0.6 mg in 1 hour (1.8
mg total)) (n=74 )
High-dose colchicine (1.2 mg po followed by 0.6 mg every
hour for 6 hours (4.8 mg total)) (n=52)
Primary Outcome ≥50% pain reduction at 24 hours without rescue medication
Results Low and high-dose colchicine were significantly superior to
placebo in reduction of pain at 24 hours (p=0.005, 0.034
respectively)
Overall adverse event rates for high-dose, low-dose, and
placebo groups were 76.9%, 36.5%, and 27.1% respectively
Terkeltaub RA, Furst DE, Bennett K, et al. High Versus Low Dosing of Oral Colchicine for Early Acute Gout Flare. Arthritis Rheum. 2010;62(4):1060-68.
Gout cdm2 2014_class version(1)
• Acute Flare:
– FDA Approved: 1.2mg po at sign of first flare,
followed in 1 hour with a single dose of 0.6mg (max
of 1.8mg)
– Real Life: FDA Approved dose (1.8mg total) then 12
hours later start prophylaxis dosing (0.6mg po daily
or BID) until the end of the gouty attack.
• Prophylaxis: 0.6mg po
once or twice daily
(max 1.2mg daily)
Arthritis Care Res. 2012:64(10);1454.
• Prophylaxis:
– No adjustments for mild-moderate renal impairment
– Dose adjust to 0.3mg po daily in severe renal impairment
– In patients on HD, the dose is 0.3mg po twice weekly
• Treatment:
– No dose adjustments for mild-severe renal impairment
– For severe renal impairment, treatment courses should not
be repeated more than once every 2 weeks
– In patients on HD, the dose is 0.6mg po once, and
treatment courses should not be repeated more than once
every 2 weeks
• Prophylaxis:
– No dose adjustment needed for mild-moderate
hepatic impairment
– Consideration to dose adjustment for severe hepatic
impairment
• Treatment:
– No dose adjustment needed for mild-severe hepatic
impairment
– In patients with severe hepatic impairment,
treatment courses should not be repeated more
than once every 2 weeks
• Adverse Effects:
– GI: Diarrhea, nausea, vomiting, abdominal
cramping/pain
– Blood dyscrasias: myelosupression, leukopenia,
granulocytopenia, thrombocytopenia, aplastic
anemia
– Neuromuscular Toxicity: myopathy,
rhabdomyolysis, muscle weakness/pain
• Contraindication: patients with renal or hepatic
impairment should not use colchicine with a p-
glycoprotein (PGP) or strong CYP3A4 inhibitorColcrys [package insert]. Philadelphia, PA: AR Scientific, Inc; 2012.
• Drug Interactions: Substrate of CYP3A4, PGP;
Induces CYP2C8, 2C9, 2E1, 3A4
– 3A4 Inhibitors
– PGP Inhibitors
– HMG Co-A Reductase Inhibitors, fibrates
Colcrys [package insert]. Philadelphia, PA: AR Scientific, Inc; 2012.
Colcrys [package insert]. Philadelphia, PA: AR Scientific, Inc; 2012.
Requires dose adjustmentsRequires dose adjustments
if currently on these meds,if currently on these meds,
or if recently (w/in the pastor if recently (w/in the past
14 days) on the meds listed14 days) on the meds listed
• Role in Therapy:
– FIRST LINE agent for acute gouty attacks w/in 36h
of attack (Evidence A)
– FIRST LINE prophylaxis in patients initiated on urate
lowering therapy for up to 6 months (Evidence A)
• Cost: 0.6mg (30 tablets) $196.15
Colcrys [package insert]. Philadelphia, PA: AR Scientific, Inc; 2012.
• MOA:
– Anakinra: Competitively inhibits IL-1 from binding to
the IL-1 type 1 receptors
– Canakinumab: Recombinant IL-1β monoclonal
antibody
• Dose:
– Anakinra: 100mg SQ daily x 3 days
– Canakinumab: 150mg SQ once
• Role in Therapy:
– Canakinumab is approved in the EU for acute gout
– FDA has not approved either in America for gout
Medication BSR EULAR ACR
Nonsteroidal
Anti-
inflammatory
Drugs (NSAIDs)
First line First Line First Line
Corticosteroid Effective
Alternative
Effective
Alternative
First Line
Colchicine Effective
Alternative
First line First Line w/in
36h on attack
onset
British Society for Rheumatology (BSR), European League Against
Rheumatism (EULAR), American College of Rheumatology (ACR)
Rheumatology. 2007;46:1372-1374. Ann Rheum Dis. 2006;65:1312-24. Arthritis Care Res. 2012:64(10);1431-1446.
• Comorbid conditions
• Effectiveness of past
treatments
• Patient preference
• Joint involvement
• Severity
• Duration of attack
• Concomitant medications
Severity of Acute Gouty Arthritis Attack (based on 0-10 visual
analog scale)
Mild ≤4
Moderate 5-6
Severe ≥7
Duration of the gouty arthritis attack since onset
Early <12h after attack onset
Well-Established 12-36h after attack onset
Late >36h after attack onset
Arthritis Care Res. 2012:64(10);1451.
Extent of acute gouty arthritis attack
One or a few small joints
1-2 large joints (ankle, knee, writs, elbow, hip, shoulder)
Polyarticular:
• ≥4 joints w/arthritis involving more than 1 region
(forefoot, midfoot, ankle/hindfoot, knee, hip,
fingers, wrist, elbow, shoulder, other)
• Acute gout attack involving 3 separate large joints
Arthritis Care Res. 2012:64(10);1451.
Adequate Response:Adequate Response:
•>20% improvement in>20% improvement in
pain score w/in 24hpain score w/in 24h
•≥50% improvement in50% improvement in
pain scorepain score ≥ 24h24h
Adequate Response:Adequate Response:
•>20% improvement in>20% improvement in
pain score w/in 24hpain score w/in 24h
•≥50% improvement in50% improvement in
pain scorepain score ≥ 24h24h
Combination Therapy:Combination Therapy:
(1)(1)NSAID + colchicineNSAID + colchicine
(2)(2)Corticosteroids +Corticosteroids +
colchicinecolchicine
(3)(3)Intra-articular steroidsIntra-articular steroids
+ anything+ anything
Combination Therapy:Combination Therapy:
(1)(1)NSAID + colchicineNSAID + colchicine
(2)(2)Corticosteroids +Corticosteroids +
colchicinecolchicine
(3)(3)Intra-articular steroidsIntra-articular steroids
+ anything+ anything
Figure 3 from Arthritis Care Res. 2012:64(10);1452.
Date of download: 5/20/2014 Copyright © 2012 McGraw-Hill Medical. All rights reserved.
Algorithm for management of an acute gout attack.
Legend:
From: Chapter 74. Gout and
Hyperuricemia
Pharmacotherapy: A
Pathophysiologic
Approach, 9e, 2014
From: Chapter 74. Gout and
Hyperuricemia
Pharmacotherapy: A
Pathophysiologic
Approach, 9e, 2014
Disease State Considerations for Selection of Acute Gout
Therapy
CKD NSAIDs, COX-2 Inhibitors, Colchicine
CHF NSAIDs, COX-2 Inhibitors,
Corticosteroids
Peptic Ulcer Disease NSAIDs, COX-2 Inhibitors,
Corticosteroids
Anticoagulation/
Antiplatelet Therapy
NSAIDs
Diabetes Corticosteroids
Infection or Infection Risk Corticosteroids
Hepatic Disease NSAIDs, COX-2 Inhibitors, Colchicine
• Self-limiting disease; ~7-10 days of treatment usually
required
• Colchicine/NSAIDs/Corticosteroids are all first line
• Select the agent based on insurance/joint
involvement/pain severity/co-morbid
conditions/concomitant medications/duration of attack
• Appropriate response is a 50% reduction in pain in 24h
• If a patient is on urate lowering therapy, it should not be
stopped during the acute attack
RT is a 48 year old male with PMH significant for
HTN, Type II DM, CHF
Patient presented to the emergency room with a 12
hour history of acute tenderness, and redness of his
left big toe.
Pain Score: 4/10 on visual analog scale
BP 178/92 mmHg in ER
145 105 22
1263.8
250
A. Naproxen 500mg po BID
B. Colchicine 1.2mg po once, then 0.6mg po one
hour later
C. Prednisone 30mg po daily
A. Naproxen 500mg po BID
B. Colchicine 1.2mg po once, then 0.6mg po one
hour later
C. Prednisone 30mg po daily
D. Intra-articular steroids
Gout cdm2 2014_class version(1)
• Presentation:
– Tophus: urate deposits
– Fingers>olecranon bursae>forearm>achilles
tendon>knees>wrists>hand
• Late complication of gout, uncommon in the
general population
• Complications:
– Joint deformities and destruction
– Pain
– Damage to surrounding tissue
Rheumatology. 2007;46:1372-1374. Ann Rheum Dis. 2006;65:1312-24. Arthritis Care Res. 2012:64(10);1431-1446.
Rheumatology. 2007;46:1372-1374. Ann Rheum Dis. 2006;65:1312-24. Arthritis Care Res. 2012:64(10);1431-1446.
Upon discussion with RT in the ER, the physician
discovers this is the third exacerbation RT has had
this year.
Is RT indicated for urate lowering therapy?
A. Yes
A. No
LOW DOSELOW DOSE
Colchicine, NSAIDs,Colchicine, NSAIDs,
CorticosteroidsCorticosteroids
Image from: www.goutrx.com
Methods Randomized, prospective, double-blind, placebo controlled trial
Treatment Arms
(n=43)
Colchicine 0.6 mg po BID (n=21)
Placebo po BID (n=22)
Primary Outcome Number of gout flares during the initiation of allopurinol
Results Reduction in the number pts with acute gout flares (33% of
colchicine pts versus 77% placebo pts (p=0.008))
Reduction in the number of pts with multiple gout flares (14% of
colchicine pts vs 63% placebo pts (p=0.004))
Borstad GC, Bryant LR, Abel MP, et al. Colchicine for Prophylaxis of Acute Flares When Initiating Allopurinol for Chronic Gouty Arthritis. J Rheumatol.
2004;31(12):2429-32.
• First Line: Colchicine or low dose NSAIDs
– Colchicine 0.6mg po daily or BID
– Low Dose NSAIDS (i.e. Naproxen 250mg po BID)
• Second Line: Corticosteroids
– Prednisone or prednisolone ≤10 po mg/day
• Role: Initiated just prior to/at start of urate lowering therapy to
prevent acute gouty attacks
Rheumatology. 2007;46:1372-1374. Ann Rheum Dis. 2006;65:1312-24. Arthritis Care Res. 2012:64(10);1431-1446.
Keep in mind toKeep in mind to
check the need forcheck the need for
GI prophylaxis withGI prophylaxis with
long term NSAIDslong term NSAIDs
Figure 5 from Arthritis Care & Research. 2012;64(10):1457.
Signs/Symptoms of Gout:Signs/Symptoms of Gout:
acute gouty arthritis in theacute gouty arthritis in the
past 3 months, presence ofpast 3 months, presence of
palpable tophus or tophi,palpable tophus or tophi,
chronic tophaceous goutychronic tophaceous gouty
arthropathy (with chronicarthropathy (with chronic
synovitis) in the past 3synovitis) in the past 3
monthsmonths
RT is a 48 year old male with PMH significant for
HTN, Type II DM, CHF
Patient presented to the emergency room with a 12
hour history of acute tenderness, and redness of his
left big toe.
Pain Score: 4/10 on visual analog scale
BP 178/92 mmHg in ER
145 105 22
1263.8
250
What prophylactic therapy would you want to use
in RT and why?
A. Colchicine
B. Naproxen
C. Prednisone
Allopurinol,Allopurinol,
Febuxostat,Febuxostat,
Probenecid,Probenecid,
Pegloticase…Pegloticase…
• Mechanism of Action: Competitively inhibits the
postsecretory renal proximal tubular
reabsorption of uric acid
• Dose:
• Avoid use with renal impairment (CrCl <50)
Probenecid. Lexi-Drugs Online, Lexi-Comp Inc. Hudson, OH. http://www.crlonline.com/crlonline. Accessed July 1, 2014.
Medication Dose
Probenecid 250mg po BID, titrated up to 500-2000mg/day
Sulfinpyrazone 50mg po BID, titrated to 100-400mg /day
• Adverse Effects:
– Cardiovascular: Flushing
– CNS: Dizziness, fever, headache
– Dermatologic: Alopecia, dermatitis, pruritis, rash
– Gastrointestinal: Anorexia, dyspepsia, GERD,
nausea
– Hematologic: Anemia, aplastic anemia, hemolytic
anemia (in G6PD deficiency), leukopenia
– Skeletal: Precipitation of acute gouty arthritis,
– Other: rash, hypersensitivity, uric acid
nephrolithiasis
• Contraindication: History of urolithiasis,
overproducers of uric acid, hypersensitivity to
probenecid, small or large dose aspirin therapy,
blood dyscrasias, <2 years of age, initiation
during an acute gout attack
• Precautions:
– Disease-related: G6PD, peptic ulcer disease, renal
impairment
– Drug-related: Methotrexate, penicillin, salicylates
• Drug Interactions:
– Salicylates
– Penicillins/cephalosporins/carbapenems
– Methotrexate
– Pegloticase
• Monitoring: serum uric acid, urine uric acid,
renal function
• Pt Education: Increase fluid intake ± urinary
alkalization
• Place in Therapy:
– “ALTERNATIVE” FIRST LINE therapy to xanthine
oxidase inhibitors (XOI)
– May be used in conjunction with XOI in patients not
fully controlled on XOI alone
• Cost: Probenecid 500mg (100 tablets) $114.56
• If patient has HTN or hyperlipidemia consider the
use of losartan and fenofibrate
• Both have uricosuric properties
• Place in Therapy: add-on to XOI in pts not
controlled on single agent
Rheumatology. 2007;46:1372-1374. Ann Rheum Dis. 2006;65:1312-24. Arthritis Care Res. 2012:64(10);1431-1446.
• Mechanism of Action: Xanthine Oxidase inhibitor
which causes reduced uric acid production
• Dosing: Start at 50-100mg po daily
– Titrate up every 2-5 weeks until a goal SUA <5-6
mg/dL
– Doses >300mg usually given in divided doses
– Max Dose=800mg/dAllopurinol. Lexi-Drugs Online, Lexi-Comp Inc. Hudson, OH. http://www.crlonline.com/crlonline. Accessed July 1, 2014.
Dalbeth N, Kumar S, Stamp L et al. Dose Adjustment of Allopurinol According to Creatinine Clearance Does Not Provide Adequate Control of Hyperuricemia in
Patients with Gout. J Rheumatol. 2006;33(8)1646-50.
Dose Adjustment of Allopurinol According to Creatinine Clearance Does Not
Provide Adequate Control of Hyperuricemia in Patients with Gout.
Methods Retrospective chart review of 250 patients attending
rheumatology clinics in South Auckland between 2001 and
2004 diagnosed with gout
Analysis Groups
(n=250)
•No allopurinol (n=23, 9.2%)
•Lower than recommended allopurinol dose (n=22, 9.7%)
•Recommended allopurinol dose (n=161, 70.9%)
•Higher than recommended allopurinol dose (n=44,
19.4%)
Inclusion Criteria American College of Rheumatology diagnostic criteria for
gout
Exclusion Criteria End stage renal failure receiving renal replacement
therapy
Dalbeth N, Kumar S, Stamp L et al. Dose Adjustment of Allopurinol According to Creatinine Clearance Does Not Provide Adequate Control of Hyperuricemia in
Patients with Gout. J Rheumatol. 2006;33(8)1646-50.
Group SUA Number of patients achieving
SUA ≤6mg/dL (0.36mmol/L)
No allopurinol 0.57 mmol/L 1/23 (4%)
Lower than
recommended
allopurinol dose
0.53 mmol/L 3/20 (15%, p=0.7 vs
recommended allopurinol dose)
Recommended
allopurinol dose
0.48 mmol/L 29/152 (19.1%)
Higher than
recommended
allopurinol dose
0.48 mmol/L 16/42 (38%, p<0.01 vs
recommended allopurinol dose)
Dalbeth N, Kumar S, Stamp L et al. Dose Adjustment of Allopurinol According to Creatinine Clearance Does Not Provide Adequate Control of Hyperuricemia in
Patients with Gout. J Rheumatol. 2006;33(8)1646-50.
Dalbeth N, Kumar S, Stamp L et al. Dose Adjustment of Allopurinol According to Creatinine Clearance Does Not Provide Adequate Control of Hyperuricemia in
Patients with Gout. J Rheumatol. 2006;33(8)1646-50.
• Renal:
– Initiation of Therapy:
• 50mg for people with CKD stage ≥ 4
• 100mg for everyone else
– Chronic Therapy:
• “Dose can be raised above 300mg daily even with renal
impairment as long as it is accompanied by adequate
education and monitoring for drug toxicity (e.g., puritis,
rash, elevated hepatic transaminases; evidence B).
• The ACR guidelines do not recommend the non-evidence
based renal dose adjustment algorithm for maintenance
dosing
• Hepatic: no dose adjustments given
Arthritis Care Res. 2012:64(10);1431-1446.
• Adverse Effects:
– Dermatologic: Rash (<1%), Allopurinol hypersensitivity
syndrome (AHS)
– Hematologic: agranulocytosis, aplastic anemia,
myeulosupression, thrombocytopenia
– Renal: renal failure (<1%)
• Contraindications: allopurinol sensitivity, concurrent
didanosine
• Drug Interaction: azathioprine, 6-mercaptourine (MUST
reduce the dose of azathioprine or 6-MP when used in
combination with allopurinol), theophylline, pegloticase,
loop/thiazide diuretics, didanosine
• Monitoring: SUA every few weeks initially, HLA-
B*5801 prior to initiation in high risk
subpopulations (Koreans with stage 3 CKD or
worse, and patients of Han Chinese or Thai
descent).
• Place in Therapy: FIRST LINE agent for chronic
gout
• Cost: 100mg (100 tablets) $24.24, 300mg (100
tablets) $60.81
• MOA: non-purine inhibitor of xanthine oxidase
(XO)
• Dosage:
– Initiate at 40mg po daily; if goal SUA is not
achieved by 2 weeks, increase to 80mg po daily
– No renal/hepatic dosage adjustment in mild-
Uloric [package insert]. Deerfield, IL: Takeda Pharmaceuticals America, Inc; 2012.
• Adverse Effects:
– Hepatic: abnormal LFTs
– GI: nausea
– Rheumatologic: Gout flares due to mobilization
– Others: arthralgia, rash
• Contraindications: Concomitant use of
azathioprine/6-mercaptopurine
• Precaution:
– Acute gout flare
– Cardiovascular events
– Hepatotoxicity
• Drug-Interactions: azathioprine/6-
mercaptourine, theophylline, pegloticase
• Monitoring: SUA after 2 weeks of treatment;
LFTs at baseline and repeated if signs of hepatic
injury
• Place in Therapy: FIRST LINE alternative to
allopurinol
• Cost:
– 40 mg (30 tablets): $256.99
– 80 mg (30 tablets): $256.99
Febuxostat Compared with Allopurinol in Patients with Hyperuricemia and Gout.
Methods Randomized, multi-center, double-blind, phase 3 trial over a 52
week period
Treatment Arms
(n=760)
•Febuxostat 80mg po daily (n=256)
•Febuxostat 120mg po daily (n=251)
•Allopurinol 300mg po daily (n=253)
Primary Outcome Serum urate concentration of <6mg/dL at the last three monthly
measurements
Secondary
Outcomes
Proportion of subjects with SUA <6mg/dL at each visit,
percentage reduction of SUA from baseline at each visit,
reduction in the incidence of gout flares, reduction in tophus
area, reduction in the number of tophi
Becker MA, Schumacher HR, Wortmann RL et al. N Engl J Med. 2005;353:2450-61.
Febuxostat Compared with Allopurinol in Patients with Hyperuricemia and Gout.
Results • Percentage of patients with SUA <6% at last 3 monthly visits
• Febuxostat 80mg (53%)*
• Febuxostat 120mg (62%)*
• Allopurinol 300mg (21%)
• Incidence of gout flares (day 1-week 8)
• Febuxostat 80mg (22%)
• Febuxostat 120mg (36%)*
• Allopurinol 300mg (21%)
Authors’ Conclusion Significantly more subjects receiving febuxostat v allopurinol
achieved the serum urate concentrations <6mg/dL
* p<0.001 compared to allopurinol
• MOA: pegylated recombinant modified
mammalian urate oxidase (uricase)
• Dose: 8mg IV infusion over 2 hours Q2 weeks;
with corticosteroid and antihistamine pre-
medicationKrystexxa [package insert]. East Brunswick, NJ: Savient Pharmaceuticals, Inc; 2012
Sundy JS, Becker MA, Baraf HS, et al. Reduction of Plasma Urate Levels Following Treatment With Multiple Doses of Pegloticase (polyethylene
Glycol-Conjugated Uricase) in Patients With Treatment-Failure Gout. Arthritis Rheum. 2008;58(9):2882-2891
• Adverse Effects: Gout flares, infusion reactions,
nausea, vomiting, bruising, nasopharyngitis,
constipation, chest pain, anaphylaxis
• Contraindication: Glucose-6-phospate
dehydrogenase deficiency
Krystexxa [package insert]. East Brunswick, NJ: Savient Pharmaceuticals, Inc; 2012
• Precautions:
– Anaphylaxis: Most often occurs within 2 hours of
the infusion; pre-medicate patients and observe
them
– Infusion reactions: Give the infusion over no less
than 120 minutes; if a reaction occurs, the infusion
should be slowed or stopped and restarted at a
lower rate
– Gout flares: Start gout flare prophylaxis with low
dose NSAID or colchicine at least 1 week prior to
starting pegloticase
– Congestive heart failure: Some patients in clinical
trials have demonstrated CHF exacerbations
• Drug Interactions: No studies of pegloticase with other
drugs have been conducted
• Monitor: SUA levels, G6PD screening prior to initiation
in high risk patient populations (African and
Mediterranean ancestry)
• Place in Therapy:
– Refractory disease
– Must have all other forms of urate lowering therapy
discontinued prior to its use
• Cost: $10026 (8mg/mL, 1mL)
XO inhibitors: allopurinol, febuxostat
XO inhibitors: allopurinol, febuxostat
Uricosurics: probenecid, losartan,
fenofibrate
Recombinant uricase
enzyme: Pegloticase
X
X
BSR EULAR ACR
Goals of
Therapy
≤ 5mg/dL ≤ 6mg/dL ≤6 mg/dL, and often ≤5
mg/dL
Prophylaxis
for Acute
Gout
Colchicine for up
to 6 months;
NSAIDs or COX-2
inhibitors in
patients who
cannot take
colchicine for up
to 6 weeks
Prophylaxis in the
first months of
urate lowering
therapy can be
achieved with
colchicine and/or
NSAID
First Line: NSAIDs or
colchicine
Second Line:
Corticosteroids
Uricosurics
(i.e.
probenecid)
Second line drug
in under-excretors
or in patients
resistant to
allopurinol
Alternative to
allopurinol in
patients with
normal renal
function
“Alternative” first line tx
w/appropriate renal
function and intolerance
or contraindication to
XOI; add-on to xanthine
oxidase inhibitors when
not fully controlled
Ann Rheum Dis. 2006;65:1312-24. Rheumatology. 2007:1-17. Arthritis Care & Research. 2012;64(10):1431-1446.
BSR EULAR ACR
Allopurinol Drug of choice;
Initial treatment
starting at 50-
100mg/day and
adjusted for renal
function if
necessary, to reach a
therapeutic target of
SUA <300μmol/L
Appropriate
therapy; start at a
low dose
(100mg/day), the
dose must be
adjusted for renal
impairment
First Line
Febuxostat Not addressed Not addressed First Line
Pegloticase Not addressed Not addressed Refractory disease
Fenofibrate/
losartan
Uricosuric Uricosuric Add on to appropriate
XOI therapy
Initiation of
Therapy
NOT during acute
attack
NOT during acute
attack
May start during
acute attack if on
prophylaxis
Ann Rheum Dis. 2006;65:1312-24. Rheumatology. 2007:1-17. Arthritis Care & Research. 2012;64(10):1431-1446.
• Historically, we do not initiate urate lowering
therapy during the acute attack due to risk of
worsening the attack
• New guideline recommendation from ACR
states “pharmacologic ULT could be started
during an acute gout attack, provided that
effective antiinflammatory management has
been instituted (level C)”
• In practice, we are still waiting 1-2 weeks after
Arthritis Care & Research. 2012;64(10):1431-1446.
Figure 3 from Arthritis Care & Research. 2012;64(10):1437.
Long TermLong Term
Monitoring:Monitoring:
•SUA Q2-5SUA Q2-5
weeks duringweeks during
ULT initiationULT initiation
•SUA Q6SUA Q6
months whenmonths when
stablestable
Table 4 from Arthritis Care & Research. 2012;64(10):1441.
• Unclear etiology of hyperuricemia
• Refractory signs/symptoms of gout
• Difficult to control and reach target SUA
• Multiple/serious medication related adverse
events
Ann Rheum Dis. 2006;65:1312-24. Rheumatology. 2007:1-17. Arthritis Care & Research. 2012;64(10):1431-1446.
• Patient should be started on prophylaxis (low dose colchicine,
NSAID, prednisone) just prior to or with the start of ULT
• XOI (allopurinol/febuxostat) are first line
• Uricosurics are usually second line or add on therapy, and should
be avoided in over-producers
• Pegloticase is used in refractory disease. All other ULT must be
stopped prior to starting pegloticase.
• Once on ULT, they should be maintained on ULT indefinitely.
Should an acute exacerbation occur, the ULT should remain on.
These are being performed at the sameThese are being performed at the same
time. We look to see if they need ULT,time. We look to see if they need ULT,
start the prophylactic therapy, and thenstart the prophylactic therapy, and then
start the ULT.start the ULT.
Date of download: 5/20/2014 Copyright © 2012 McGraw-Hill Medical. All rights reserved.
Algorithm for management of hyperuricemia in gout.
Legend:
From: Chapter 74. Gout and Hyperuricemia
Pharmacotherapy: A Pathophysiologic Approach, 9e, 2014
From: Chapter 74. Gout and Hyperuricemia
Pharmacotherapy: A Pathophysiologic Approach, 9e, 2014
Upon discussion with RT in the ER, the physician
discovers this is the third exacerbation RT has had
this year. He has only been given is 1.8mg total of
colchicine thus far.
What is the next thing that should be given to RT?
A.Nothing—he is still in an acute exacerbation
B.Probenecid 250mg po BID
C.Colchicine 0.6mg po BID
D.Naproxen 250mg po BID
What Urate Lowering Therapy would you want to
start in RT, and why?
A. Allopurinol
B. Febuxostat
C. Probenecid
D. Nothing
When would you want to start urate lowering
therapy in RT?
A.Immediately, he is already on prophylactic
therapy
B.After he demonstrates appropriate response to
his current treatment, which can double as
prophylactic therapy
C.1-2 weeks after his current flare subsides while he
is on concurrent prophylactic therapy
• Should always be considered alone or in
combination with pharmacologic therapy
• Smoking cessation
• Patient Education
• Exercise
• Weight loss
• Avoidance of risk factors (dietary, medication)
~10-18%~10-18%
decrease indecrease in
SUASUA
Rheumatology. 2007;46:1372-1374. Ann Rheum Dis. 2006;65:1312-24. Arthritis Care Res. 2012:64(10);1431-1446.
Avoid Limit Encourage
• Organ Meats high in purine
(eg, sweetbreads, liver,
kidney)
• Serving Sizes of:
• Beef, lamb, pork
• Seafood with high purine
content (eg, sardines,
shellfish)
• Low-fat or non-fat dairy
products
• High fructose corn syrup-
sweetened sodas, other
beverages, or foods
• Servings of naturally sweet
fruit juices
• Table sugar, and sweetened
beverages and desserts
• Table salt, including in
sauces and gravies
• Vegetables
• Alcohol overuse (defined as
more than 2 servings/day
for males and 1 serving/day
for females) in ALL gout
patients
• Any alcohol use in gout
during periods of frequent
gout attacks, or advanced
gout with poor control
• Alcohol (particularly beer,
but also wine and spirits) in
all gout patients
Figure 4 from Arthritis Care & Research. 2012;64(10):1439.
Given the following patient case, identify the
options for non-pharmacologic treatment of
gout:
• RT is 6’4” and 240lbs
• He exercises some mornings during the week
• RT generally has coffee for breakfast, a pepsi
with a roast beef sandwich for lunch, and
scallops or steak with vegetables and beer for
dinner.
• RT is 6’4” and 240lbs
• Current Medications: metoprolol XL 50mg po
daily, aspirin 81mg po daily, lisinopril 20mg po
daily, metformin 1000mg po BID, furosemide
40mg po daily
What medication risk factors does RT current
have, and can we change any of his
medications?
• Royal Pains: Season 2 Episode 10 (available on
Netflix)
– 13:20-14:33
– 28:37-29:45
– 33:25-35
• What would be in your differential diagnosis for
this patient?
• What signs/symptoms of gout does this patient
present with?
• Is this a typical acute gouty presentation?
Gout cdm2 2014_class version(1)
A.Treatment plan is appropriate
B. Treatment plan is inappropriate
Gout cdm2 2014_class version(1)
A. Start the allopurinol as planned.
B. Treat the acute attack, then start allopurinol
C. Treat the acute attack, then start low dose
colchicine and allopurinol concurrently
D. Treat the acute attack and modify risk factors
• Acute
– Non-Pharmacologic: rest, ice
– Pharmacologic:
• NSAIDs, colchicine, corticosteroids to treat (~7-10 day)
• CONTINUE ULT during the acute gouty attack!
• Evaluate if indicated for ULT: Tophi, CKD Stage
≥2, Urolithiasis, ≥2 acute gout attacks/year
• Chronic
– Non-Pharmacologic: diet, fluids,
weight loss, smoking cessation,
avoidance of risk factors
– Pharmacologic:
• Prophylactic therapy: LOW DOSE
colchicine, NSAIDs, corticosteroids
during initiation or just prior to initiation
of ULT
• Urate lowering therapy (ULT) for
indefinite duration dosed to SUA
– Allopurinol/febuxostat> probenecid
– Probenecid/losartan/fenofibrate may
be add on therapy
– Pegloticase last line for refractory
cases
Neoni T. Gout. N Engl J Med. 2011;364:443-52.
Gout cdm2 2014_class version(1)
• We probably wont have time to work on these
during class, but Elizabeth Akselrod, PharmD
Class of 2015, and I made some practice cases
and questions for you to work on at home while
studying for the quiz/exam.
• I will post our keys to the questions at the end of
my version of the ppt slides after our lecture.
1. Which of the following is the goal of urate
lowering therapy?
A. SUA <6mg/dL
B. SUA <4 mg/dL
C. SUA <7mg/dL
D. Lower the SUA until the tophi resolve
2. Which medication(s) work on xanthine oxidase
A. Allopurinol
B. Febuxostat
C. Pegloticase
D. Probenecid
E. Both A and B
3) There is new evidence to support dosing
allopurinol to a target serum uric acid, instead of
renal dose adjustment
a) True
b) False
3) Low dose colchicine is less effective than high
dose colchicine
a) True
b) False
5) Patients being treated for gout should receive
pharmacologic treatment only
a) True
b) False
6) When initiating urate lowering therapy, which
of the following can be given to prevent acute
gout attacks
a) NSAIDs
b) Colchicine
c) Corticosteroids
d) All of the above
• MC is a 21 yoF who presents to her PCP for an annual checkup. Her parents have
been concerned because lately she hasn’t been taking good care of herself. She’s
been staying up late, eating lots of processed foods, drinking excessive alcohol since
her 21st
birthday, and has developed a “strange obsession with sticking her tongue
out at people.” She presents with no symptoms and simply wants to “get this
appointment over with so she can go party in the USA.”
• PMH: N/A
• SH: Drinks 3-4 mixed drinks per day, quit smoking 4 weeks ago (1 pack per day)
• Vitals: 98.6 F, BP 115/76, HR 80, RR 18, O2 Sat 98
• Medications: Women’s MVI, daily laxative use to help with weight loss
• Abnormal labs: SUA 7.2 mg/dL
• List all MC’s risk factors
• Does MC need treatment for gout? If yes, what would you recommend? If no,
explain.
• RW (prefers to be called Fat Amy) is a 28 yoF who was shooting a film in Hollywood when all of a
sudden she felt pain in her right big toe which was red and inflamed. She presented to the ER 3
hours later and stated that her pain was a 4/10. She has no history of acute gouty attacks.
• PMH: Type II DM, Obesity, GERD (no ulcers)
• SH: 2 beers per night
• SUA: 6.5 mg/dL; no tophi present
• CrCl: 60 mL/min
• What treatment options can you recommend for RW?
• Does RW need chronic gout treatment? Prophylaxis? Explain.
• Provide one counseling point using patient-specific language.
• TS is a 59 yoM who presents to the ER for the 4th
time this year with an acute
gouty attack. He rates his pain as a 6/10 and says it started 24 hours ago. His
history is significant for HTN (refuses to take his Lisinopril because it makes
him cough), DMT2, s/p MI in 2009, migraines, gouty attacks with tophi, and
s/p vehicle accident with fractured femur. SUA: 8. G6PD (-) and HLA-B*5801
(-).
• Recommend a plan for TS. Include drug name, dose, route and duration.
• If the patient was HLA-B*5801 (+), which medication would you have to
avoid?
• Which medication(s) is contraindicated in G6PD deficiency?
• SN is a 63 yoM who has no significant PMH except has been taking Allopurinol 300
mg PO daily for 1 year due to frequent gouty attacks starting in 2005. At his annual
physical, the doctor notes that SN has chronic knee pain and an SUA of 8.5 mg/dL.
The doctor asks for your advice. Since you’re a gout expert, you know exactly what
options are available and you recommend:
• List 3 non-pharmacologic options to educate SN about:
• HR is a 45 yoF who has had breakthrough gouty
attacks despite Allopurinol treatment. Her PMH
is significant for HTN, DMT2, hyperlipidemia,
s/p kidney donation in 2010, and recent start on
Azathioprine for RA. She tells the doctor that
she has heard of an alternative to Allopurinol
that also doesn’t require renal dosing. What is
this drug? Is HR eligible to receive this
medication? Why/why not?
• Dr. Sampson is a new resident and wants to start
a patient on Allopurinol for chronic gout
treatment. He asks you for help to make sure the
patient is eligible to take this medication. What
test should be ordered prior to starting the
patient on Allopurinol?
• Which medications should you beware in the
patient’s profile that interacts with Allopurinol?
• TY comes into your pharmacy to pick up his newly
prescribed probenecid. He has never heard of this
medication and knows nothing about it. You check his
profile and confirm that he doesn’t have renal
impairment and you approve the order. What
important points should you make TY aware of?
• RC 67 year old African American male comes is a
regular patient at your pharmacy. His past
medical history includes Hypertension, diabetes,
and atrial fibrillation. His current medication list
includes: chlorthalidone 25mg po daily,
metformin 1000mg po BID, aspirin 81mg po
daily, verapamil 80mg po TID, and warfarin 5mg
po daily.
• VS: within normal limits; BG: 150 (fasting this
AM)
• He comes to your pharmacy with the following
prescription. What would you want to do?
Colcrys 1.2mg po once followed by
0.6mg 1 hour later, then 12 hours
later start 0.6mg po BID x 7 days.
RC 67
7/6/14

More Related Content

What's hot

vancomycin protocol
vancomycin protocolvancomycin protocol
vancomycin protocolMarwa gamal
 
Antihyperlipidemics1
Antihyperlipidemics1Antihyperlipidemics1
Antihyperlipidemics1Dr Shah Murad
 
FDA's advice on_acetaminophen_(paracetamol)_associated serious skin reactions
FDA's advice on_acetaminophen_(paracetamol)_associated serious skin reactionsFDA's advice on_acetaminophen_(paracetamol)_associated serious skin reactions
FDA's advice on_acetaminophen_(paracetamol)_associated serious skin reactionsNaina Mohamed, PhD
 
Non-steroidal anti-inflammatory drugs (NSAIDs) Pharmacology review
Non-steroidal anti-inflammatory drugs (NSAIDs)  Pharmacology reviewNon-steroidal anti-inflammatory drugs (NSAIDs)  Pharmacology review
Non-steroidal anti-inflammatory drugs (NSAIDs) Pharmacology reviewPharmacy Universe
 
Pharmacotherapy, Management of Hypertension, JNC 8 guidelines
Pharmacotherapy, Management of Hypertension, JNC 8 guidelinesPharmacotherapy, Management of Hypertension, JNC 8 guidelines
Pharmacotherapy, Management of Hypertension, JNC 8 guidelinesankitamishra1402
 
Pharmacology of Nervous System
Pharmacology of Nervous SystemPharmacology of Nervous System
Pharmacology of Nervous Systemmharun5
 
Diabetic Kidney Disease (DKD) : 2022 update
 Diabetic Kidney Disease (DKD) : 2022 update  Diabetic Kidney Disease (DKD) : 2022 update
Diabetic Kidney Disease (DKD) : 2022 update Malsawmkima Chhakchhuak
 
The organs damages as side effects of NSAIDs
The organs damages as side effects of NSAIDsThe organs damages as side effects of NSAIDs
The organs damages as side effects of NSAIDsMuhamed Al Rohani
 
PH1.3 Enumerate and identify drug formulations and drug delivery systems
PH1.3 Enumerate and identify drug formulations and drug delivery systemsPH1.3 Enumerate and identify drug formulations and drug delivery systems
PH1.3 Enumerate and identify drug formulations and drug delivery systemsPKGupta8
 
Anti hypertensives and diuretics drugs - pharmacology
Anti hypertensives and diuretics drugs - pharmacology Anti hypertensives and diuretics drugs - pharmacology
Anti hypertensives and diuretics drugs - pharmacology Areej Abu Hanieh
 
Pharm d research project
Pharm d research projectPharm d research project
Pharm d research projectSai Kumar
 
Essential drug concept and rational use of medicines
Essential drug concept and rational use of medicinesEssential drug concept and rational use of medicines
Essential drug concept and rational use of medicinesPravin Prasad
 
Pathophysiology and Drug Therapy of Migraine
Pathophysiology and Drug Therapy of MigrainePathophysiology and Drug Therapy of Migraine
Pathophysiology and Drug Therapy of MigraineSawsan Aboul-Fotouh
 

What's hot (20)

vancomycin protocol
vancomycin protocolvancomycin protocol
vancomycin protocol
 
Antihyperlipidemics1
Antihyperlipidemics1Antihyperlipidemics1
Antihyperlipidemics1
 
FDA's advice on_acetaminophen_(paracetamol)_associated serious skin reactions
FDA's advice on_acetaminophen_(paracetamol)_associated serious skin reactionsFDA's advice on_acetaminophen_(paracetamol)_associated serious skin reactions
FDA's advice on_acetaminophen_(paracetamol)_associated serious skin reactions
 
Non-steroidal anti-inflammatory drugs (NSAIDs) Pharmacology review
Non-steroidal anti-inflammatory drugs (NSAIDs)  Pharmacology reviewNon-steroidal anti-inflammatory drugs (NSAIDs)  Pharmacology review
Non-steroidal anti-inflammatory drugs (NSAIDs) Pharmacology review
 
Antihypertensive drugs 2015-16
Antihypertensive drugs 2015-16Antihypertensive drugs 2015-16
Antihypertensive drugs 2015-16
 
Pharmacotherapy, Management of Hypertension, JNC 8 guidelines
Pharmacotherapy, Management of Hypertension, JNC 8 guidelinesPharmacotherapy, Management of Hypertension, JNC 8 guidelines
Pharmacotherapy, Management of Hypertension, JNC 8 guidelines
 
Pharmacology of Nervous System
Pharmacology of Nervous SystemPharmacology of Nervous System
Pharmacology of Nervous System
 
Roxadustat_CKD
Roxadustat_CKDRoxadustat_CKD
Roxadustat_CKD
 
L7
L7L7
L7
 
Dapagliflozin- a novel SGLT2 inhibitor
Dapagliflozin- a novel SGLT2 inhibitorDapagliflozin- a novel SGLT2 inhibitor
Dapagliflozin- a novel SGLT2 inhibitor
 
Diabetic Kidney Disease (DKD) : 2022 update
 Diabetic Kidney Disease (DKD) : 2022 update  Diabetic Kidney Disease (DKD) : 2022 update
Diabetic Kidney Disease (DKD) : 2022 update
 
The organs damages as side effects of NSAIDs
The organs damages as side effects of NSAIDsThe organs damages as side effects of NSAIDs
The organs damages as side effects of NSAIDs
 
PH1.3 Enumerate and identify drug formulations and drug delivery systems
PH1.3 Enumerate and identify drug formulations and drug delivery systemsPH1.3 Enumerate and identify drug formulations and drug delivery systems
PH1.3 Enumerate and identify drug formulations and drug delivery systems
 
Drug interactions
Drug interactionsDrug interactions
Drug interactions
 
Anti hypertensives and diuretics drugs - pharmacology
Anti hypertensives and diuretics drugs - pharmacology Anti hypertensives and diuretics drugs - pharmacology
Anti hypertensives and diuretics drugs - pharmacology
 
Polypharmacy
PolypharmacyPolypharmacy
Polypharmacy
 
Pharm d research project
Pharm d research projectPharm d research project
Pharm d research project
 
Polypharmacy
PolypharmacyPolypharmacy
Polypharmacy
 
Essential drug concept and rational use of medicines
Essential drug concept and rational use of medicinesEssential drug concept and rational use of medicines
Essential drug concept and rational use of medicines
 
Pathophysiology and Drug Therapy of Migraine
Pathophysiology and Drug Therapy of MigrainePathophysiology and Drug Therapy of Migraine
Pathophysiology and Drug Therapy of Migraine
 

Viewers also liked (6)

Gout
Gout Gout
Gout
 
Gout Disease
Gout DiseaseGout Disease
Gout Disease
 
Gout.
Gout.Gout.
Gout.
 
Gout
GoutGout
Gout
 
Gout
GoutGout
Gout
 
Gout presentation
Gout presentationGout presentation
Gout presentation
 

Similar to Gout cdm2 2014_class version(1)

Sulfad trial germany 2012
Sulfad trial germany 2012Sulfad trial germany 2012
Sulfad trial germany 2012tratpharma
 
Sulfad trial germany 2012
Sulfad trial germany 2012Sulfad trial germany 2012
Sulfad trial germany 2012solbzrah
 
C13 nice type 2 diabetes 2008
C13 nice type 2 diabetes 2008C13 nice type 2 diabetes 2008
C13 nice type 2 diabetes 2008Diabetes for all
 
Gout ii acr 2012
Gout ii   acr 2012Gout ii   acr 2012
Gout ii acr 2012Hummd Mdhum
 
Feb 2015 journal watch with links
Feb 2015 journal watch with linksFeb 2015 journal watch with links
Feb 2015 journal watch with linkskatejohnpunag
 
Study and solution to high blood pressure.
Study and solution to high blood pressure.Study and solution to high blood pressure.
Study and solution to high blood pressure.Chekwbe
 
Jnc 8 jama dic 2013
Jnc 8 jama dic 2013Jnc 8 jama dic 2013
Jnc 8 jama dic 2013raularnez2
 
VIII reporte hipertension
VIII reporte hipertensionVIII reporte hipertension
VIII reporte hipertensionmelissalazaro1
 
Samir rafla jnc 8-2014 evidence-based guideline for the management of high bl...
Samir rafla jnc 8-2014 evidence-based guideline for the management of high bl...Samir rafla jnc 8-2014 evidence-based guideline for the management of high bl...
Samir rafla jnc 8-2014 evidence-based guideline for the management of high bl...Alexandria University, Egypt
 
The effect of vitamin D add-on therapy on the improvement of.pdf
The effect of vitamin D add-on therapy on the improvement of.pdfThe effect of vitamin D add-on therapy on the improvement of.pdf
The effect of vitamin D add-on therapy on the improvement of.pdfFinnyOktaria
 
Synthetic Drugs/Hormones - Boon or Bane- Concept of Dooshivisha and Gara Visha
Synthetic Drugs/Hormones - Boon or Bane- Concept of Dooshivisha and Gara VishaSynthetic Drugs/Hormones - Boon or Bane- Concept of Dooshivisha and Gara Visha
Synthetic Drugs/Hormones - Boon or Bane- Concept of Dooshivisha and Gara VishaIJARIIT
 
2018 Q4 Rheumatoid Arthritis Management.pptx
2018 Q4 Rheumatoid Arthritis Management.pptx2018 Q4 Rheumatoid Arthritis Management.pptx
2018 Q4 Rheumatoid Arthritis Management.pptxjavakhirMuradov
 

Similar to Gout cdm2 2014_class version(1) (20)

Sulfad trial germany 2012
Sulfad trial germany 2012Sulfad trial germany 2012
Sulfad trial germany 2012
 
Sulfad trial germany 2012
Sulfad trial germany 2012Sulfad trial germany 2012
Sulfad trial germany 2012
 
C13 nice type 2 diabetes 2008
C13 nice type 2 diabetes 2008C13 nice type 2 diabetes 2008
C13 nice type 2 diabetes 2008
 
Gout ii acr 2012
Gout ii   acr 2012Gout ii   acr 2012
Gout ii acr 2012
 
Feb 2015 journal watch with links
Feb 2015 journal watch with linksFeb 2015 journal watch with links
Feb 2015 journal watch with links
 
Study and solution to high blood pressure.
Study and solution to high blood pressure.Study and solution to high blood pressure.
Study and solution to high blood pressure.
 
Jnc 8 jama dic 2013
Jnc 8 jama dic 2013Jnc 8 jama dic 2013
Jnc 8 jama dic 2013
 
Jnc viii
Jnc viiiJnc viii
Jnc viii
 
VIII reporte hipertension
VIII reporte hipertensionVIII reporte hipertension
VIII reporte hipertension
 
ManejoJnc 8
ManejoJnc 8ManejoJnc 8
ManejoJnc 8
 
Jsc130010 jnc
Jsc130010  jncJsc130010  jnc
Jsc130010 jnc
 
8 jnc
8 jnc8 jnc
8 jnc
 
Samir rafla jnc 8-2014 evidence-based guideline for the management of high bl...
Samir rafla jnc 8-2014 evidence-based guideline for the management of high bl...Samir rafla jnc 8-2014 evidence-based guideline for the management of high bl...
Samir rafla jnc 8-2014 evidence-based guideline for the management of high bl...
 
The effect of vitamin D add-on therapy on the improvement of.pdf
The effect of vitamin D add-on therapy on the improvement of.pdfThe effect of vitamin D add-on therapy on the improvement of.pdf
The effect of vitamin D add-on therapy on the improvement of.pdf
 
Surviving sepsis campaign
Surviving sepsis campaignSurviving sepsis campaign
Surviving sepsis campaign
 
Jnc8
Jnc8Jnc8
Jnc8
 
Synthetic Drugs/Hormones - Boon or Bane- Concept of Dooshivisha and Gara Visha
Synthetic Drugs/Hormones - Boon or Bane- Concept of Dooshivisha and Gara VishaSynthetic Drugs/Hormones - Boon or Bane- Concept of Dooshivisha and Gara Visha
Synthetic Drugs/Hormones - Boon or Bane- Concept of Dooshivisha and Gara Visha
 
Diabesity with Sharon Weinstein
Diabesity with Sharon WeinsteinDiabesity with Sharon Weinstein
Diabesity with Sharon Weinstein
 
OA.ppt
OA.pptOA.ppt
OA.ppt
 
2018 Q4 Rheumatoid Arthritis Management.pptx
2018 Q4 Rheumatoid Arthritis Management.pptx2018 Q4 Rheumatoid Arthritis Management.pptx
2018 Q4 Rheumatoid Arthritis Management.pptx
 

Recently uploaded

FINAL PROJECT IN EMPOWERMENT TECHNOLOGIES 11
FINAL PROJECT IN EMPOWERMENT TECHNOLOGIES  11FINAL PROJECT IN EMPOWERMENT TECHNOLOGIES  11
FINAL PROJECT IN EMPOWERMENT TECHNOLOGIES 11crzljavier
 
Pharmacovigilance audits inspections.pptx
Pharmacovigilance audits inspections.pptxPharmacovigilance audits inspections.pptx
Pharmacovigilance audits inspections.pptxCliniminds India
 
Living Well Every Day: Lyons Wellness Practice | Nurtures Your Complete Health
Living Well Every Day: Lyons Wellness Practice | Nurtures Your Complete HealthLiving Well Every Day: Lyons Wellness Practice | Nurtures Your Complete Health
Living Well Every Day: Lyons Wellness Practice | Nurtures Your Complete HealthLyons Health
 
2024 Compliatric Webianr Series - Contracts and MOUs from a HRSA Perspective.pdf
2024 Compliatric Webianr Series - Contracts and MOUs from a HRSA Perspective.pdf2024 Compliatric Webianr Series - Contracts and MOUs from a HRSA Perspective.pdf
2024 Compliatric Webianr Series - Contracts and MOUs from a HRSA Perspective.pdfCompliatric Where Compliance Happens
 
Empathy Is a Stress Response - Choose Compassion instead
Empathy Is a Stress Response - Choose Compassion insteadEmpathy Is a Stress Response - Choose Compassion instead
Empathy Is a Stress Response - Choose Compassion insteadAlex Clapson
 
Identifying Signs of Mental Health Presentation (1).pptx
Identifying Signs of Mental Health Presentation (1).pptxIdentifying Signs of Mental Health Presentation (1).pptx
Identifying Signs of Mental Health Presentation (1).pptxsandhulove46637
 
Three Keys to a Successful Margin: Charges, Costs, and Labor
Three Keys to a Successful Margin: Charges, Costs, and LaborThree Keys to a Successful Margin: Charges, Costs, and Labor
Three Keys to a Successful Margin: Charges, Costs, and LaborHealth Catalyst
 
LARYNGEAL CANCER.pptx Prepared by Neha Kewat
LARYNGEAL CANCER.pptx  Prepared by Neha KewatLARYNGEAL CANCER.pptx  Prepared by Neha Kewat
LARYNGEAL CANCER.pptx Prepared by Neha KewatNehaKewat
 
NEONATAL RESPIRATORY CARE FROM A PHYSIO POV.pptx
NEONATAL RESPIRATORY CARE FROM A PHYSIO POV.pptxNEONATAL RESPIRATORY CARE FROM A PHYSIO POV.pptx
NEONATAL RESPIRATORY CARE FROM A PHYSIO POV.pptxHanineHassan2
 
Understanding Warts and Moles: Differences, Types, and Common Locations
Understanding Warts and Moles: Differences, Types, and Common LocationsUnderstanding Warts and Moles: Differences, Types, and Common Locations
Understanding Warts and Moles: Differences, Types, and Common LocationsNeha Sharma
 
CECT NECK NECK ANGIOGRAPHY CAROTID ANGIOGRAPHY
CECT NECK NECK ANGIOGRAPHY CAROTID ANGIOGRAPHYCECT NECK NECK ANGIOGRAPHY CAROTID ANGIOGRAPHY
CECT NECK NECK ANGIOGRAPHY CAROTID ANGIOGRAPHYRMC
 
ARTHRITIS.pptx Prepared by monika gopal Tutor
ARTHRITIS.pptx Prepared  by monika gopal TutorARTHRITIS.pptx Prepared  by monika gopal Tutor
ARTHRITIS.pptx Prepared by monika gopal TutorNehaKewat
 
EYE CANCER.pptx prepared by Neha kewat digital learning
EYE CANCER.pptx prepared by  Neha kewat digital learningEYE CANCER.pptx prepared by  Neha kewat digital learning
EYE CANCER.pptx prepared by Neha kewat digital learningNehaKewat
 
Assisted Living Care Residency - PapayaCare
Assisted Living Care Residency - PapayaCareAssisted Living Care Residency - PapayaCare
Assisted Living Care Residency - PapayaCareratilalthakkar704
 
Introduction to Evaluation and Skin Benefits
Introduction to Evaluation and Skin BenefitsIntroduction to Evaluation and Skin Benefits
Introduction to Evaluation and Skin Benefitssahilgabhane29
 
Hematinics and Erythropoietin- Pharmacology of Hematinics
Hematinics and Erythropoietin- Pharmacology of HematinicsHematinics and Erythropoietin- Pharmacology of Hematinics
Hematinics and Erythropoietin- Pharmacology of Hematinicsnetraangadi2
 
ACCA Version of AI & Healthcare: An Overview for the Curious
ACCA Version of AI & Healthcare: An Overview for the CuriousACCA Version of AI & Healthcare: An Overview for the Curious
ACCA Version of AI & Healthcare: An Overview for the CuriousKR_Barker
 
person with disability and pwd act ppt.pptx
person with disability and pwd act ppt.pptxperson with disability and pwd act ppt.pptx
person with disability and pwd act ppt.pptxMUKESH PADMANABHAN
 
Health literacies in marginalised communities LILAC 24.pptx
Health literacies in marginalised communities LILAC 24.pptxHealth literacies in marginalised communities LILAC 24.pptx
Health literacies in marginalised communities LILAC 24.pptxPamela McKinney
 

Recently uploaded (20)

FINAL PROJECT IN EMPOWERMENT TECHNOLOGIES 11
FINAL PROJECT IN EMPOWERMENT TECHNOLOGIES  11FINAL PROJECT IN EMPOWERMENT TECHNOLOGIES  11
FINAL PROJECT IN EMPOWERMENT TECHNOLOGIES 11
 
Pharmacovigilance audits inspections.pptx
Pharmacovigilance audits inspections.pptxPharmacovigilance audits inspections.pptx
Pharmacovigilance audits inspections.pptx
 
Living Well Every Day: Lyons Wellness Practice | Nurtures Your Complete Health
Living Well Every Day: Lyons Wellness Practice | Nurtures Your Complete HealthLiving Well Every Day: Lyons Wellness Practice | Nurtures Your Complete Health
Living Well Every Day: Lyons Wellness Practice | Nurtures Your Complete Health
 
2024 Compliatric Webianr Series - Contracts and MOUs from a HRSA Perspective.pdf
2024 Compliatric Webianr Series - Contracts and MOUs from a HRSA Perspective.pdf2024 Compliatric Webianr Series - Contracts and MOUs from a HRSA Perspective.pdf
2024 Compliatric Webianr Series - Contracts and MOUs from a HRSA Perspective.pdf
 
Empathy Is a Stress Response - Choose Compassion instead
Empathy Is a Stress Response - Choose Compassion insteadEmpathy Is a Stress Response - Choose Compassion instead
Empathy Is a Stress Response - Choose Compassion instead
 
Identifying Signs of Mental Health Presentation (1).pptx
Identifying Signs of Mental Health Presentation (1).pptxIdentifying Signs of Mental Health Presentation (1).pptx
Identifying Signs of Mental Health Presentation (1).pptx
 
Three Keys to a Successful Margin: Charges, Costs, and Labor
Three Keys to a Successful Margin: Charges, Costs, and LaborThree Keys to a Successful Margin: Charges, Costs, and Labor
Three Keys to a Successful Margin: Charges, Costs, and Labor
 
LARYNGEAL CANCER.pptx Prepared by Neha Kewat
LARYNGEAL CANCER.pptx  Prepared by Neha KewatLARYNGEAL CANCER.pptx  Prepared by Neha Kewat
LARYNGEAL CANCER.pptx Prepared by Neha Kewat
 
NEONATAL RESPIRATORY CARE FROM A PHYSIO POV.pptx
NEONATAL RESPIRATORY CARE FROM A PHYSIO POV.pptxNEONATAL RESPIRATORY CARE FROM A PHYSIO POV.pptx
NEONATAL RESPIRATORY CARE FROM A PHYSIO POV.pptx
 
Understanding Warts and Moles: Differences, Types, and Common Locations
Understanding Warts and Moles: Differences, Types, and Common LocationsUnderstanding Warts and Moles: Differences, Types, and Common Locations
Understanding Warts and Moles: Differences, Types, and Common Locations
 
CECT NECK NECK ANGIOGRAPHY CAROTID ANGIOGRAPHY
CECT NECK NECK ANGIOGRAPHY CAROTID ANGIOGRAPHYCECT NECK NECK ANGIOGRAPHY CAROTID ANGIOGRAPHY
CECT NECK NECK ANGIOGRAPHY CAROTID ANGIOGRAPHY
 
ARTHRITIS.pptx Prepared by monika gopal Tutor
ARTHRITIS.pptx Prepared  by monika gopal TutorARTHRITIS.pptx Prepared  by monika gopal Tutor
ARTHRITIS.pptx Prepared by monika gopal Tutor
 
EYE CANCER.pptx prepared by Neha kewat digital learning
EYE CANCER.pptx prepared by  Neha kewat digital learningEYE CANCER.pptx prepared by  Neha kewat digital learning
EYE CANCER.pptx prepared by Neha kewat digital learning
 
Assisted Living Care Residency - PapayaCare
Assisted Living Care Residency - PapayaCareAssisted Living Care Residency - PapayaCare
Assisted Living Care Residency - PapayaCare
 
Painting Rats White Angers Them to No End
Painting Rats White Angers Them to No EndPainting Rats White Angers Them to No End
Painting Rats White Angers Them to No End
 
Introduction to Evaluation and Skin Benefits
Introduction to Evaluation and Skin BenefitsIntroduction to Evaluation and Skin Benefits
Introduction to Evaluation and Skin Benefits
 
Hematinics and Erythropoietin- Pharmacology of Hematinics
Hematinics and Erythropoietin- Pharmacology of HematinicsHematinics and Erythropoietin- Pharmacology of Hematinics
Hematinics and Erythropoietin- Pharmacology of Hematinics
 
ACCA Version of AI & Healthcare: An Overview for the Curious
ACCA Version of AI & Healthcare: An Overview for the CuriousACCA Version of AI & Healthcare: An Overview for the Curious
ACCA Version of AI & Healthcare: An Overview for the Curious
 
person with disability and pwd act ppt.pptx
person with disability and pwd act ppt.pptxperson with disability and pwd act ppt.pptx
person with disability and pwd act ppt.pptx
 
Health literacies in marginalised communities LILAC 24.pptx
Health literacies in marginalised communities LILAC 24.pptxHealth literacies in marginalised communities LILAC 24.pptx
Health literacies in marginalised communities LILAC 24.pptx
 

Gout cdm2 2014_class version(1)

  • 1. GoutGout Alexa Carlson, PharmD, BCPS Northeastern University
  • 2. 1. Define and explain commonly used medical terminology and abbreviations within the context of a patient case. 2. Describe and correlate signs and symptoms, epidemiology, risk factors, pathogenesis, natural history of disease, clinical course, etiology, and prevention and treatment of commonly encountered diseases. 3. Apply scientific knowledge and principles of medicinal chemistry, pharmacology, pharmaceutics, pharmacogenomics and pharmacokinetics to the design of rational therapeutic strategies 4. Identify evidence based, patient specific therapeutic goals and outcome measures for a patient started on a drug therapy regimen. 5. Apply evidenced based medicine to compare and contrast therapeutic options for a given medical problem and/or disease state in terms of safety, efficacy (including clinical evidence), ease of administration, cost, and other patient specific factors. 6. Design and defend a patient-specific therapeutic regimen, which is safe, efficacious and practical. For each drug selected, include drug name (generic and brand) dose, route, frequency and time of administration, and duration of therapy when applicable. 7. Predict the influence of selected drugs on a patient’s other diseases or problems and outline which agents should be avoided or monitored more carefully.
  • 3. Neogi T. Gout. N Engl J Med. 2011:364(5):443-452. Khanna D, Fitzgerald JD, Khanna PP, et al. 2012 American College of Rheumatology Guidelines for Management of Gout. Part 1: Systematic Nonpharmacologic and Pharmacologic Therapeutic Approaches to Hyperuricemia. Arthritis Care & Research. 2012:64(10);1431-1446. Khanna D, Khanna PP, Fitzgerald JD, et al. 2012 American College of Rheumatology Guidelines for Management of Gout. Part 2: Therapy and Antiinflammatory Prophylaxis of Acute Gouty Arthritis. Arthritis Care & Research. 2012:64(10);1447- 1461. Jordan DM, Cameron JS, Snaith M, et al. British Society for Rheumatology Guideline for the Management of Gout. Rheumatology. 2007;46:1372-1374. Zhang W, Doherty M, Pascual E, et al. EULAR evidence based recommendations for gout. Part I: Report of a task force of the standing committee for international clinical studies including therapeutics (ESCISIT). Ann Rheum Dis. 2006l65:1301-1311. Zhang W, Doherty M, Bardin T, et al. EULAR evidence based recommendations for gout. Part II: Management. Report of a task force of the EULAR Standing Committee for International Clinical Studies Including Therapeutics (ESCISIT). Ann Rheum Dis. 2006;65:1312-1324. Roddy E, Zhang W, Doherty M. Concordance of the management of chronic gout in a UK primary-care population with the EULAR gout recommendations. Ann Rheum Dis. 2007;66:1311-1315.
  • 4. Gout is the most common cause of inflammatory arthritis in the US Substantial gaps in the management of gout Significant lack of patient education and adherence to management
  • 5. Hyperuricemia: elevated serum uric acid Tophus: a calculus containing sodium urate that develops around fibrous tissue around joints, typically in patients with gout Podagra: a painful condition of the big toe caused by gout Uricase: An enzyme that oxidatively degrade uric acid, thereby catalyzing conversion to soluble allantoin, which is much more soluble than uric acid. This enzyme is found in most animals, but not in humans. Uricosuric medications: Medications administered to increase the elimination of uric acid.
  • 6. Elevated serum uric acid (SUA) >7mg/dL (416μmol/L) at 37°C for men >6mg/dL (357μmol/L) at 37°C for women Considered to be the principle cause of gouty arthritis Not all patients with hyperuricemia develop acute gout flares or chronic gouty complications
  • 7. Clinical spectrum of disease: Hyperuricemia Recurrent acute arthritis attacks due to monosodium urate (MSU) crystals in synovial fluid Deposition of MSU into articular and extra-articular space (tophi) Interstitial renal disease Uric acid nephrolithiasis Primarily caused by elevated SUA levels, but can occur in patients with normal SUA levels. http://www.healthinplainenglish.com/health /musculoskeletal/gout/ http://www.assh.org/Public/HandConditions/Pag es/GoutandPseudogout.aspx
  • 8. “Disease of Kings” Alexander the Great King Henry VIII Benjamin Franklin Alexander Hamilton Voltaire Health.com List of 8 people with gout: http://www.health.com/health/gallery/0,,2045 1892,00.html Image from: antabusediaries.blogspot.com
  • 9. Increasing prevalence of gout Developed Countries > Developing Countries 3.9% of adults in the US (~8.3 million people) $3.9 million in annual physician visits Richette P, Bardin T. Gout. Lancet. 2010;375:318-28.
  • 10. Date of download: 5/20/2014 Copyright © 2012 McGraw-Hill Medical. All rights reserved. Purine metabolism. (HGPRT, hypoxanthine-guanine phosphoribosyltransferase; PRPP, phosphoribosyl pyrophosphate.) Legend: From: Chapter 74. Gout and Hyperuricemia Pharmacotherapy: A Pathophysiologic Approach, 9e, 2014
  • 11. Uric acid is the end product of purine metabolism
  • 12. Elevated uric acid can occur from over-production or under-excretion of uric acid Over-production: Less common Genetic abnormalities in enzymes related purine metabolism Increased phosphoribosyl pyrophosphate synthetase (PRPP) Decreased hypoxanthine-guanine phosphoribosylthransferase (HGPRT) Increased cell turnover (cytotoxic chemotherapy, malignancies) Under-excretion: More common (~90%) Decline in urinary excretion of uric acid Can determine by measuring urine uric acid >600mg/24h over-production <600mg/24h under-excretion
  • 14. A. All patients with gout with progress to chronic tophaceous gout B. All patients with gout have hyperuricemia C. Patients with gout will have recurrent acute attacks separated by intercritical periods D. Gout is primarily due to the overproduction of uric acid
  • 15. Increasing age Sex (Male > Female) Hyperuricemia Obesity High Blood Pressure Injury Fasting Recent surgery Foods/Drinks Medications Medical Conditions Genetics
  • 16. Foods high in purines Foods and drinks with high fructose corn syrup Alcohol Picture taken from: aldrodriguezliverfoundation.com
  • 17. Diuretics (thiazide and loop diuretics) Cyclosporine and tacrolimus Low dose salicylates (<2g/day) Levodopa Pyrazinamide Cytotoxic chemotherapy Cyclosporine Nicotinic Acid (Niacin) Ethambutol
  • 18. Chronic renal insufficiency Hypothyroidism Renal Transplant Hypertension Coronary heart disease Myeloproliferative disorders Lymphoproliferative disorders Lead toxicity Starvation Acromegaly Down’s syndrome Hypoxanthine-guanine phosphoribosyltransferase deficiency Diabetes mellitus Volume depletion Ketoacidosis Lactic Acidosis Hyper/Hypo-parathyroidism Sarcoidosis Preeclampsia Starvation Heart Failure Acute alcoholism Phosphoribosylpyrophosphate synthetase overactivity Pernicious anemia Psoriasis
  • 19. RT is a 48 year old male PMH: HTN, Type II DM, CHF SH: 5 pack-year history, drinks 2 beers per night Current Medications: metoprolol XL 50mg po daily, aspirin 81mg po daily, lisinopril 20mg po daily, metformin 1000mg po BID, furosemide 40mg po daily
  • 20. A. One B. Two C. Three D. Four
  • 21. Symptoms:Symptoms: •Rapid onsetRapid onset •Over night or after a trigger (alcohol,Over night or after a trigger (alcohol, meats, diuretics)meats, diuretics) •Excruciating painExcruciating pain Signs:Signs: •Monoarticular arthritis early on (mayMonoarticular arthritis early on (may become polyarticular in late gout)become polyarticular in late gout) •Often occurs in a joint in the lowerOften occurs in a joint in the lower extremityextremity •Redness/Swelling/Inflammation/WarmtRedness/Swelling/Inflammation/Warmt h of the jointh of the joint •FeverFever •Elevated serum uric acid leukocytosisElevated serum uric acid leukocytosis •Monosodium urate cyrstals in synovialMonosodium urate cyrstals in synovial fluidfluid
  • 23. • Differential Diagnosis: – Pseudogout (calcium pyrophosphate crystals) – Septic arthritis – Rheumatoid arthritis – Trauma • Diagnosis: – Evidence of monosodium urate (MSU) crystals by joint aspiration is gold standard for definitive diagnosis – Often made on as a clinical diagnosis
  • 24. Zhang W, Doherty M, Pascual E, et al. EULAR Evidence Based Recommendations for Gout. Part I: Diagnosis. Report of a Task Force of the Standing Committee for International Clinical Studies Including Therapeutics (ESCISIT). Ann Rheum Dis. 2006;65:1301-11.
  • 26. • Level A: Supported by multiple (ie, more than one) randomized clinical trials or meta-analyses • Level B: Derived from a single randomized trial, or nonrandomized studies • Level C: Consensus opinion of experts, case studies, or standard-of-care
  • 29. • Acute flares are self-limiting • Treatment objective is rapid relief of symptoms • All therapy should be initiated within 24h of gout symptom onset and continued for 1-2 weeks • Do NOT discontinue urate lowering therapy in an acute attack Rheumatology. 2007;46:1372-1374. Ann Rheum Dis. 2006;65:1312-24. Arthritis Care Res. 2012:64(10);1431-1446.
  • 30. Ann Rheum Dis. 2006;65:1312-24.
  • 31. • Mechanism of Action: Peripheral inhibition of COX leading to inhibition of prostaglandin synthesis • Dosing: NSAID Typical Regimen Etodolac 300mg Q12h Fenoprofen 300-600mg Q6-8h Ibuprofen 800mg Q6h Indomethacin 25-50mg Q6h x 3d, then taper to Q12h x 4-7d Ketoprofen 75mg Q6h Naproxen 500mg Q12h x 3d, then 250-500mg daily x 4-7d Piroxicam 20mg daily or divided Q12h Sulindac 200mg Q12h x 7-10d
  • 32. • Adverse Effects: increased BP, sodium and water retention, gastritis, GI bleeding • Contraindications/Precautions: Ann Rheum Dis. 2006:65:;312-24. NEJM. 2011:364(5);443-452.
  • 33. • FDA-Approved: indomethacin, naproxen, sulindac • NO NSAID has been shown to be superior to another • Role in Therapy: – FIRST LINE for acute gouty attacks w/o contraindication at high doses FIRST LINE alternative option to colchicine for prophylaxis
  • 34. • Mechanism of Action: anti-inflammatory • Dosing: Corticosteroid Typical Regimen Prednisone 0.5 mg/kg/day x5-10d, then stop (Evidence A) <OR> 0.5mg/kg/day x2-5d, then taper for 7-10d, then stop (Evidence C) Methylprednisolone Methylprednisolone oral dose pack (Evidence C) Methylprednisolone IM 100-150mg daily x1-2d Triamcinolone IM 60mg once, then oral prednisone (Evidence C) Triamcinolone intra-articular 10-40mg (large joints) or 5-20mg (small joints) once (Evidence B)
  • 35. • Adverse Effects: hyperglycemia, leukocytosis, fluid retention, impaired wound healing, GI upset, insomnia • Precautions: Infection, DM, peptic ulcer disease • Role in Therapy: – FIRST LINE therapy in acute gout – Consider intra-articular corticosteroids in patients with gout affecting 1-2 large joints – SECOND LINE therapy in prophylaxis to NSAIDs and colchicineJanssens HJEM, Janssen M, van de Lisdonk EH, et al. Use of Oral Prednisolone or Naproxen for the Treatment of Gout Arthritis: a Double-Blind, Randomized Equivalence Trial. Lancet. 2008;371:1854-60.
  • 36. Use of Oral Prednisolone or Naproxen for the Treatment of Gout Arthritis: a Double-Blind, Randomized Equivalence Trial Methods Randomized, double-blind, double-dummy, active-comparator, controlled trial in monoarticular gout to assess equivalency Intervention • Naproxen 500mg po BID x 5 days (n=59) • Prednisolone 35mg po daily x 5 days (n=59) Primary Outcomes Pain in the affected joint measured by the 100mm visual analogue scale Results Reduction in mean pain score on visual analogue scale on day 4: •44.7mm prednisolone •46mm naproxen •Difference 1.3mm (95% CI -9.8-7.1) Relief of symptoms on day 4: •Clinically significant improvements: 80% prednisolone and 87% naproxen patients •Complete relief of symptoms: 22% prednisolone and 17% naproxen patients Janssens HJEM, Janssen M, van de Lisdonk EH, et al. Use of Oral Prednisolone or Naproxen for the Treatment of Gout Arthritis: a Double-Blind, Randomized Equivalence Trial. Lancet. 2008;371:1854-60.
  • 37. Janssens HJEM, Janssen M, van de Lisdonk EH, et al. Use of Oral Prednisolone or Naproxen for the Treatment of Gout Arthritis: a Double-Blind, Randomized Equivalence Trial. Lancet. 2008;371:1854-60. Janssens HJEM, Janssen M, van de Lisdonk EH, et al. Use of Oral Prednisolone or Naproxen for the Treatment of Gout Arthritis: a Double-Blind, Randomized Equivalence Trial. Lancet. 2008;371:1854-60.
  • 38. • Mechanism of Action: – May interfere with the intracellular assembly of the inflammasome complex present in neutrophils and monocytes that mediates the activation of interlukin-1β – Disruption of cytoskeletal functions through inhibition of β-tubulin polymerization into microtubules which prevents the activation, degranulation, and migration of neutrophils Colcrys [package insert]. Philadelphia, PA: AR Scientific, Inc; 2012.
  • 39. Methods Multicenter, randomized, double-blind, placebo- controlled, parallel-group study Treatment Arms (n=184) Placebo (n=59) Low-dose colchicine (1.2 mg po with 0.6 mg in 1 hour (1.8 mg total)) (n=74 ) High-dose colchicine (1.2 mg po followed by 0.6 mg every hour for 6 hours (4.8 mg total)) (n=52) Primary Outcome ≥50% pain reduction at 24 hours without rescue medication Results Low and high-dose colchicine were significantly superior to placebo in reduction of pain at 24 hours (p=0.005, 0.034 respectively) Overall adverse event rates for high-dose, low-dose, and placebo groups were 76.9%, 36.5%, and 27.1% respectively Terkeltaub RA, Furst DE, Bennett K, et al. High Versus Low Dosing of Oral Colchicine for Early Acute Gout Flare. Arthritis Rheum. 2010;62(4):1060-68.
  • 41. • Acute Flare: – FDA Approved: 1.2mg po at sign of first flare, followed in 1 hour with a single dose of 0.6mg (max of 1.8mg) – Real Life: FDA Approved dose (1.8mg total) then 12 hours later start prophylaxis dosing (0.6mg po daily or BID) until the end of the gouty attack. • Prophylaxis: 0.6mg po once or twice daily (max 1.2mg daily) Arthritis Care Res. 2012:64(10);1454.
  • 42. • Prophylaxis: – No adjustments for mild-moderate renal impairment – Dose adjust to 0.3mg po daily in severe renal impairment – In patients on HD, the dose is 0.3mg po twice weekly • Treatment: – No dose adjustments for mild-severe renal impairment – For severe renal impairment, treatment courses should not be repeated more than once every 2 weeks – In patients on HD, the dose is 0.6mg po once, and treatment courses should not be repeated more than once every 2 weeks
  • 43. • Prophylaxis: – No dose adjustment needed for mild-moderate hepatic impairment – Consideration to dose adjustment for severe hepatic impairment • Treatment: – No dose adjustment needed for mild-severe hepatic impairment – In patients with severe hepatic impairment, treatment courses should not be repeated more than once every 2 weeks
  • 44. • Adverse Effects: – GI: Diarrhea, nausea, vomiting, abdominal cramping/pain – Blood dyscrasias: myelosupression, leukopenia, granulocytopenia, thrombocytopenia, aplastic anemia – Neuromuscular Toxicity: myopathy, rhabdomyolysis, muscle weakness/pain • Contraindication: patients with renal or hepatic impairment should not use colchicine with a p- glycoprotein (PGP) or strong CYP3A4 inhibitorColcrys [package insert]. Philadelphia, PA: AR Scientific, Inc; 2012.
  • 45. • Drug Interactions: Substrate of CYP3A4, PGP; Induces CYP2C8, 2C9, 2E1, 3A4 – 3A4 Inhibitors – PGP Inhibitors – HMG Co-A Reductase Inhibitors, fibrates Colcrys [package insert]. Philadelphia, PA: AR Scientific, Inc; 2012.
  • 46. Colcrys [package insert]. Philadelphia, PA: AR Scientific, Inc; 2012. Requires dose adjustmentsRequires dose adjustments if currently on these meds,if currently on these meds, or if recently (w/in the pastor if recently (w/in the past 14 days) on the meds listed14 days) on the meds listed
  • 47. • Role in Therapy: – FIRST LINE agent for acute gouty attacks w/in 36h of attack (Evidence A) – FIRST LINE prophylaxis in patients initiated on urate lowering therapy for up to 6 months (Evidence A) • Cost: 0.6mg (30 tablets) $196.15 Colcrys [package insert]. Philadelphia, PA: AR Scientific, Inc; 2012.
  • 48. • MOA: – Anakinra: Competitively inhibits IL-1 from binding to the IL-1 type 1 receptors – Canakinumab: Recombinant IL-1β monoclonal antibody • Dose: – Anakinra: 100mg SQ daily x 3 days – Canakinumab: 150mg SQ once • Role in Therapy: – Canakinumab is approved in the EU for acute gout – FDA has not approved either in America for gout
  • 49. Medication BSR EULAR ACR Nonsteroidal Anti- inflammatory Drugs (NSAIDs) First line First Line First Line Corticosteroid Effective Alternative Effective Alternative First Line Colchicine Effective Alternative First line First Line w/in 36h on attack onset British Society for Rheumatology (BSR), European League Against Rheumatism (EULAR), American College of Rheumatology (ACR) Rheumatology. 2007;46:1372-1374. Ann Rheum Dis. 2006;65:1312-24. Arthritis Care Res. 2012:64(10);1431-1446.
  • 50. • Comorbid conditions • Effectiveness of past treatments • Patient preference • Joint involvement • Severity • Duration of attack • Concomitant medications
  • 51. Severity of Acute Gouty Arthritis Attack (based on 0-10 visual analog scale) Mild ≤4 Moderate 5-6 Severe ≥7 Duration of the gouty arthritis attack since onset Early <12h after attack onset Well-Established 12-36h after attack onset Late >36h after attack onset Arthritis Care Res. 2012:64(10);1451.
  • 52. Extent of acute gouty arthritis attack One or a few small joints 1-2 large joints (ankle, knee, writs, elbow, hip, shoulder) Polyarticular: • ≥4 joints w/arthritis involving more than 1 region (forefoot, midfoot, ankle/hindfoot, knee, hip, fingers, wrist, elbow, shoulder, other) • Acute gout attack involving 3 separate large joints Arthritis Care Res. 2012:64(10);1451.
  • 53. Adequate Response:Adequate Response: •>20% improvement in>20% improvement in pain score w/in 24hpain score w/in 24h •≥50% improvement in50% improvement in pain scorepain score ≥ 24h24h Adequate Response:Adequate Response: •>20% improvement in>20% improvement in pain score w/in 24hpain score w/in 24h •≥50% improvement in50% improvement in pain scorepain score ≥ 24h24h Combination Therapy:Combination Therapy: (1)(1)NSAID + colchicineNSAID + colchicine (2)(2)Corticosteroids +Corticosteroids + colchicinecolchicine (3)(3)Intra-articular steroidsIntra-articular steroids + anything+ anything Combination Therapy:Combination Therapy: (1)(1)NSAID + colchicineNSAID + colchicine (2)(2)Corticosteroids +Corticosteroids + colchicinecolchicine (3)(3)Intra-articular steroidsIntra-articular steroids + anything+ anything Figure 3 from Arthritis Care Res. 2012:64(10);1452.
  • 54. Date of download: 5/20/2014 Copyright © 2012 McGraw-Hill Medical. All rights reserved. Algorithm for management of an acute gout attack. Legend: From: Chapter 74. Gout and Hyperuricemia Pharmacotherapy: A Pathophysiologic Approach, 9e, 2014 From: Chapter 74. Gout and Hyperuricemia Pharmacotherapy: A Pathophysiologic Approach, 9e, 2014
  • 55. Disease State Considerations for Selection of Acute Gout Therapy CKD NSAIDs, COX-2 Inhibitors, Colchicine CHF NSAIDs, COX-2 Inhibitors, Corticosteroids Peptic Ulcer Disease NSAIDs, COX-2 Inhibitors, Corticosteroids Anticoagulation/ Antiplatelet Therapy NSAIDs Diabetes Corticosteroids Infection or Infection Risk Corticosteroids Hepatic Disease NSAIDs, COX-2 Inhibitors, Colchicine
  • 56. • Self-limiting disease; ~7-10 days of treatment usually required • Colchicine/NSAIDs/Corticosteroids are all first line • Select the agent based on insurance/joint involvement/pain severity/co-morbid conditions/concomitant medications/duration of attack • Appropriate response is a 50% reduction in pain in 24h • If a patient is on urate lowering therapy, it should not be stopped during the acute attack
  • 57. RT is a 48 year old male with PMH significant for HTN, Type II DM, CHF Patient presented to the emergency room with a 12 hour history of acute tenderness, and redness of his left big toe. Pain Score: 4/10 on visual analog scale BP 178/92 mmHg in ER 145 105 22 1263.8 250
  • 58. A. Naproxen 500mg po BID B. Colchicine 1.2mg po once, then 0.6mg po one hour later C. Prednisone 30mg po daily
  • 59. A. Naproxen 500mg po BID B. Colchicine 1.2mg po once, then 0.6mg po one hour later C. Prednisone 30mg po daily D. Intra-articular steroids
  • 61. • Presentation: – Tophus: urate deposits – Fingers>olecranon bursae>forearm>achilles tendon>knees>wrists>hand • Late complication of gout, uncommon in the general population • Complications: – Joint deformities and destruction – Pain – Damage to surrounding tissue
  • 62. Rheumatology. 2007;46:1372-1374. Ann Rheum Dis. 2006;65:1312-24. Arthritis Care Res. 2012:64(10);1431-1446.
  • 63. Rheumatology. 2007;46:1372-1374. Ann Rheum Dis. 2006;65:1312-24. Arthritis Care Res. 2012:64(10);1431-1446.
  • 64. Upon discussion with RT in the ER, the physician discovers this is the third exacerbation RT has had this year. Is RT indicated for urate lowering therapy? A. Yes A. No
  • 65. LOW DOSELOW DOSE Colchicine, NSAIDs,Colchicine, NSAIDs, CorticosteroidsCorticosteroids
  • 67. Methods Randomized, prospective, double-blind, placebo controlled trial Treatment Arms (n=43) Colchicine 0.6 mg po BID (n=21) Placebo po BID (n=22) Primary Outcome Number of gout flares during the initiation of allopurinol Results Reduction in the number pts with acute gout flares (33% of colchicine pts versus 77% placebo pts (p=0.008)) Reduction in the number of pts with multiple gout flares (14% of colchicine pts vs 63% placebo pts (p=0.004)) Borstad GC, Bryant LR, Abel MP, et al. Colchicine for Prophylaxis of Acute Flares When Initiating Allopurinol for Chronic Gouty Arthritis. J Rheumatol. 2004;31(12):2429-32.
  • 68. • First Line: Colchicine or low dose NSAIDs – Colchicine 0.6mg po daily or BID – Low Dose NSAIDS (i.e. Naproxen 250mg po BID) • Second Line: Corticosteroids – Prednisone or prednisolone ≤10 po mg/day • Role: Initiated just prior to/at start of urate lowering therapy to prevent acute gouty attacks Rheumatology. 2007;46:1372-1374. Ann Rheum Dis. 2006;65:1312-24. Arthritis Care Res. 2012:64(10);1431-1446. Keep in mind toKeep in mind to check the need forcheck the need for GI prophylaxis withGI prophylaxis with long term NSAIDslong term NSAIDs
  • 69. Figure 5 from Arthritis Care & Research. 2012;64(10):1457. Signs/Symptoms of Gout:Signs/Symptoms of Gout: acute gouty arthritis in theacute gouty arthritis in the past 3 months, presence ofpast 3 months, presence of palpable tophus or tophi,palpable tophus or tophi, chronic tophaceous goutychronic tophaceous gouty arthropathy (with chronicarthropathy (with chronic synovitis) in the past 3synovitis) in the past 3 monthsmonths
  • 70. RT is a 48 year old male with PMH significant for HTN, Type II DM, CHF Patient presented to the emergency room with a 12 hour history of acute tenderness, and redness of his left big toe. Pain Score: 4/10 on visual analog scale BP 178/92 mmHg in ER 145 105 22 1263.8 250
  • 71. What prophylactic therapy would you want to use in RT and why? A. Colchicine B. Naproxen C. Prednisone
  • 73. • Mechanism of Action: Competitively inhibits the postsecretory renal proximal tubular reabsorption of uric acid • Dose: • Avoid use with renal impairment (CrCl <50) Probenecid. Lexi-Drugs Online, Lexi-Comp Inc. Hudson, OH. http://www.crlonline.com/crlonline. Accessed July 1, 2014. Medication Dose Probenecid 250mg po BID, titrated up to 500-2000mg/day Sulfinpyrazone 50mg po BID, titrated to 100-400mg /day
  • 74. • Adverse Effects: – Cardiovascular: Flushing – CNS: Dizziness, fever, headache – Dermatologic: Alopecia, dermatitis, pruritis, rash – Gastrointestinal: Anorexia, dyspepsia, GERD, nausea – Hematologic: Anemia, aplastic anemia, hemolytic anemia (in G6PD deficiency), leukopenia – Skeletal: Precipitation of acute gouty arthritis, – Other: rash, hypersensitivity, uric acid nephrolithiasis
  • 75. • Contraindication: History of urolithiasis, overproducers of uric acid, hypersensitivity to probenecid, small or large dose aspirin therapy, blood dyscrasias, <2 years of age, initiation during an acute gout attack • Precautions: – Disease-related: G6PD, peptic ulcer disease, renal impairment – Drug-related: Methotrexate, penicillin, salicylates
  • 76. • Drug Interactions: – Salicylates – Penicillins/cephalosporins/carbapenems – Methotrexate – Pegloticase • Monitoring: serum uric acid, urine uric acid, renal function • Pt Education: Increase fluid intake ± urinary alkalization
  • 77. • Place in Therapy: – “ALTERNATIVE” FIRST LINE therapy to xanthine oxidase inhibitors (XOI) – May be used in conjunction with XOI in patients not fully controlled on XOI alone • Cost: Probenecid 500mg (100 tablets) $114.56
  • 78. • If patient has HTN or hyperlipidemia consider the use of losartan and fenofibrate • Both have uricosuric properties • Place in Therapy: add-on to XOI in pts not controlled on single agent Rheumatology. 2007;46:1372-1374. Ann Rheum Dis. 2006;65:1312-24. Arthritis Care Res. 2012:64(10);1431-1446.
  • 79. • Mechanism of Action: Xanthine Oxidase inhibitor which causes reduced uric acid production • Dosing: Start at 50-100mg po daily – Titrate up every 2-5 weeks until a goal SUA <5-6 mg/dL – Doses >300mg usually given in divided doses – Max Dose=800mg/dAllopurinol. Lexi-Drugs Online, Lexi-Comp Inc. Hudson, OH. http://www.crlonline.com/crlonline. Accessed July 1, 2014. Dalbeth N, Kumar S, Stamp L et al. Dose Adjustment of Allopurinol According to Creatinine Clearance Does Not Provide Adequate Control of Hyperuricemia in Patients with Gout. J Rheumatol. 2006;33(8)1646-50.
  • 80. Dose Adjustment of Allopurinol According to Creatinine Clearance Does Not Provide Adequate Control of Hyperuricemia in Patients with Gout. Methods Retrospective chart review of 250 patients attending rheumatology clinics in South Auckland between 2001 and 2004 diagnosed with gout Analysis Groups (n=250) •No allopurinol (n=23, 9.2%) •Lower than recommended allopurinol dose (n=22, 9.7%) •Recommended allopurinol dose (n=161, 70.9%) •Higher than recommended allopurinol dose (n=44, 19.4%) Inclusion Criteria American College of Rheumatology diagnostic criteria for gout Exclusion Criteria End stage renal failure receiving renal replacement therapy Dalbeth N, Kumar S, Stamp L et al. Dose Adjustment of Allopurinol According to Creatinine Clearance Does Not Provide Adequate Control of Hyperuricemia in Patients with Gout. J Rheumatol. 2006;33(8)1646-50.
  • 81. Group SUA Number of patients achieving SUA ≤6mg/dL (0.36mmol/L) No allopurinol 0.57 mmol/L 1/23 (4%) Lower than recommended allopurinol dose 0.53 mmol/L 3/20 (15%, p=0.7 vs recommended allopurinol dose) Recommended allopurinol dose 0.48 mmol/L 29/152 (19.1%) Higher than recommended allopurinol dose 0.48 mmol/L 16/42 (38%, p<0.01 vs recommended allopurinol dose) Dalbeth N, Kumar S, Stamp L et al. Dose Adjustment of Allopurinol According to Creatinine Clearance Does Not Provide Adequate Control of Hyperuricemia in Patients with Gout. J Rheumatol. 2006;33(8)1646-50.
  • 82. Dalbeth N, Kumar S, Stamp L et al. Dose Adjustment of Allopurinol According to Creatinine Clearance Does Not Provide Adequate Control of Hyperuricemia in Patients with Gout. J Rheumatol. 2006;33(8)1646-50.
  • 83. • Renal: – Initiation of Therapy: • 50mg for people with CKD stage ≥ 4 • 100mg for everyone else – Chronic Therapy: • “Dose can be raised above 300mg daily even with renal impairment as long as it is accompanied by adequate education and monitoring for drug toxicity (e.g., puritis, rash, elevated hepatic transaminases; evidence B). • The ACR guidelines do not recommend the non-evidence based renal dose adjustment algorithm for maintenance dosing • Hepatic: no dose adjustments given Arthritis Care Res. 2012:64(10);1431-1446.
  • 84. • Adverse Effects: – Dermatologic: Rash (<1%), Allopurinol hypersensitivity syndrome (AHS) – Hematologic: agranulocytosis, aplastic anemia, myeulosupression, thrombocytopenia – Renal: renal failure (<1%) • Contraindications: allopurinol sensitivity, concurrent didanosine • Drug Interaction: azathioprine, 6-mercaptourine (MUST reduce the dose of azathioprine or 6-MP when used in combination with allopurinol), theophylline, pegloticase, loop/thiazide diuretics, didanosine
  • 85. • Monitoring: SUA every few weeks initially, HLA- B*5801 prior to initiation in high risk subpopulations (Koreans with stage 3 CKD or worse, and patients of Han Chinese or Thai descent). • Place in Therapy: FIRST LINE agent for chronic gout • Cost: 100mg (100 tablets) $24.24, 300mg (100 tablets) $60.81
  • 86. • MOA: non-purine inhibitor of xanthine oxidase (XO) • Dosage: – Initiate at 40mg po daily; if goal SUA is not achieved by 2 weeks, increase to 80mg po daily – No renal/hepatic dosage adjustment in mild- Uloric [package insert]. Deerfield, IL: Takeda Pharmaceuticals America, Inc; 2012.
  • 87. • Adverse Effects: – Hepatic: abnormal LFTs – GI: nausea – Rheumatologic: Gout flares due to mobilization – Others: arthralgia, rash • Contraindications: Concomitant use of azathioprine/6-mercaptopurine
  • 88. • Precaution: – Acute gout flare – Cardiovascular events – Hepatotoxicity • Drug-Interactions: azathioprine/6- mercaptourine, theophylline, pegloticase
  • 89. • Monitoring: SUA after 2 weeks of treatment; LFTs at baseline and repeated if signs of hepatic injury • Place in Therapy: FIRST LINE alternative to allopurinol • Cost: – 40 mg (30 tablets): $256.99 – 80 mg (30 tablets): $256.99
  • 90. Febuxostat Compared with Allopurinol in Patients with Hyperuricemia and Gout. Methods Randomized, multi-center, double-blind, phase 3 trial over a 52 week period Treatment Arms (n=760) •Febuxostat 80mg po daily (n=256) •Febuxostat 120mg po daily (n=251) •Allopurinol 300mg po daily (n=253) Primary Outcome Serum urate concentration of <6mg/dL at the last three monthly measurements Secondary Outcomes Proportion of subjects with SUA <6mg/dL at each visit, percentage reduction of SUA from baseline at each visit, reduction in the incidence of gout flares, reduction in tophus area, reduction in the number of tophi Becker MA, Schumacher HR, Wortmann RL et al. N Engl J Med. 2005;353:2450-61.
  • 91. Febuxostat Compared with Allopurinol in Patients with Hyperuricemia and Gout. Results • Percentage of patients with SUA <6% at last 3 monthly visits • Febuxostat 80mg (53%)* • Febuxostat 120mg (62%)* • Allopurinol 300mg (21%) • Incidence of gout flares (day 1-week 8) • Febuxostat 80mg (22%) • Febuxostat 120mg (36%)* • Allopurinol 300mg (21%) Authors’ Conclusion Significantly more subjects receiving febuxostat v allopurinol achieved the serum urate concentrations <6mg/dL * p<0.001 compared to allopurinol
  • 92. • MOA: pegylated recombinant modified mammalian urate oxidase (uricase) • Dose: 8mg IV infusion over 2 hours Q2 weeks; with corticosteroid and antihistamine pre- medicationKrystexxa [package insert]. East Brunswick, NJ: Savient Pharmaceuticals, Inc; 2012 Sundy JS, Becker MA, Baraf HS, et al. Reduction of Plasma Urate Levels Following Treatment With Multiple Doses of Pegloticase (polyethylene Glycol-Conjugated Uricase) in Patients With Treatment-Failure Gout. Arthritis Rheum. 2008;58(9):2882-2891
  • 93. • Adverse Effects: Gout flares, infusion reactions, nausea, vomiting, bruising, nasopharyngitis, constipation, chest pain, anaphylaxis • Contraindication: Glucose-6-phospate dehydrogenase deficiency Krystexxa [package insert]. East Brunswick, NJ: Savient Pharmaceuticals, Inc; 2012
  • 94. • Precautions: – Anaphylaxis: Most often occurs within 2 hours of the infusion; pre-medicate patients and observe them – Infusion reactions: Give the infusion over no less than 120 minutes; if a reaction occurs, the infusion should be slowed or stopped and restarted at a lower rate – Gout flares: Start gout flare prophylaxis with low dose NSAID or colchicine at least 1 week prior to starting pegloticase – Congestive heart failure: Some patients in clinical trials have demonstrated CHF exacerbations
  • 95. • Drug Interactions: No studies of pegloticase with other drugs have been conducted • Monitor: SUA levels, G6PD screening prior to initiation in high risk patient populations (African and Mediterranean ancestry) • Place in Therapy: – Refractory disease – Must have all other forms of urate lowering therapy discontinued prior to its use • Cost: $10026 (8mg/mL, 1mL)
  • 96. XO inhibitors: allopurinol, febuxostat XO inhibitors: allopurinol, febuxostat Uricosurics: probenecid, losartan, fenofibrate Recombinant uricase enzyme: Pegloticase X X
  • 97. BSR EULAR ACR Goals of Therapy ≤ 5mg/dL ≤ 6mg/dL ≤6 mg/dL, and often ≤5 mg/dL Prophylaxis for Acute Gout Colchicine for up to 6 months; NSAIDs or COX-2 inhibitors in patients who cannot take colchicine for up to 6 weeks Prophylaxis in the first months of urate lowering therapy can be achieved with colchicine and/or NSAID First Line: NSAIDs or colchicine Second Line: Corticosteroids Uricosurics (i.e. probenecid) Second line drug in under-excretors or in patients resistant to allopurinol Alternative to allopurinol in patients with normal renal function “Alternative” first line tx w/appropriate renal function and intolerance or contraindication to XOI; add-on to xanthine oxidase inhibitors when not fully controlled Ann Rheum Dis. 2006;65:1312-24. Rheumatology. 2007:1-17. Arthritis Care & Research. 2012;64(10):1431-1446.
  • 98. BSR EULAR ACR Allopurinol Drug of choice; Initial treatment starting at 50- 100mg/day and adjusted for renal function if necessary, to reach a therapeutic target of SUA <300μmol/L Appropriate therapy; start at a low dose (100mg/day), the dose must be adjusted for renal impairment First Line Febuxostat Not addressed Not addressed First Line Pegloticase Not addressed Not addressed Refractory disease Fenofibrate/ losartan Uricosuric Uricosuric Add on to appropriate XOI therapy Initiation of Therapy NOT during acute attack NOT during acute attack May start during acute attack if on prophylaxis Ann Rheum Dis. 2006;65:1312-24. Rheumatology. 2007:1-17. Arthritis Care & Research. 2012;64(10):1431-1446.
  • 99. • Historically, we do not initiate urate lowering therapy during the acute attack due to risk of worsening the attack • New guideline recommendation from ACR states “pharmacologic ULT could be started during an acute gout attack, provided that effective antiinflammatory management has been instituted (level C)” • In practice, we are still waiting 1-2 weeks after Arthritis Care & Research. 2012;64(10):1431-1446.
  • 100. Figure 3 from Arthritis Care & Research. 2012;64(10):1437. Long TermLong Term Monitoring:Monitoring: •SUA Q2-5SUA Q2-5 weeks duringweeks during ULT initiationULT initiation •SUA Q6SUA Q6 months whenmonths when stablestable
  • 101. Table 4 from Arthritis Care & Research. 2012;64(10):1441.
  • 102. • Unclear etiology of hyperuricemia • Refractory signs/symptoms of gout • Difficult to control and reach target SUA • Multiple/serious medication related adverse events Ann Rheum Dis. 2006;65:1312-24. Rheumatology. 2007:1-17. Arthritis Care & Research. 2012;64(10):1431-1446.
  • 103. • Patient should be started on prophylaxis (low dose colchicine, NSAID, prednisone) just prior to or with the start of ULT • XOI (allopurinol/febuxostat) are first line • Uricosurics are usually second line or add on therapy, and should be avoided in over-producers • Pegloticase is used in refractory disease. All other ULT must be stopped prior to starting pegloticase. • Once on ULT, they should be maintained on ULT indefinitely. Should an acute exacerbation occur, the ULT should remain on.
  • 104. These are being performed at the sameThese are being performed at the same time. We look to see if they need ULT,time. We look to see if they need ULT, start the prophylactic therapy, and thenstart the prophylactic therapy, and then start the ULT.start the ULT.
  • 105. Date of download: 5/20/2014 Copyright © 2012 McGraw-Hill Medical. All rights reserved. Algorithm for management of hyperuricemia in gout. Legend: From: Chapter 74. Gout and Hyperuricemia Pharmacotherapy: A Pathophysiologic Approach, 9e, 2014 From: Chapter 74. Gout and Hyperuricemia Pharmacotherapy: A Pathophysiologic Approach, 9e, 2014
  • 106. Upon discussion with RT in the ER, the physician discovers this is the third exacerbation RT has had this year. He has only been given is 1.8mg total of colchicine thus far. What is the next thing that should be given to RT? A.Nothing—he is still in an acute exacerbation B.Probenecid 250mg po BID C.Colchicine 0.6mg po BID D.Naproxen 250mg po BID
  • 107. What Urate Lowering Therapy would you want to start in RT, and why? A. Allopurinol B. Febuxostat C. Probenecid D. Nothing
  • 108. When would you want to start urate lowering therapy in RT? A.Immediately, he is already on prophylactic therapy B.After he demonstrates appropriate response to his current treatment, which can double as prophylactic therapy C.1-2 weeks after his current flare subsides while he is on concurrent prophylactic therapy
  • 109. • Should always be considered alone or in combination with pharmacologic therapy • Smoking cessation • Patient Education • Exercise • Weight loss • Avoidance of risk factors (dietary, medication) ~10-18%~10-18% decrease indecrease in SUASUA
  • 110. Rheumatology. 2007;46:1372-1374. Ann Rheum Dis. 2006;65:1312-24. Arthritis Care Res. 2012:64(10);1431-1446.
  • 111. Avoid Limit Encourage • Organ Meats high in purine (eg, sweetbreads, liver, kidney) • Serving Sizes of: • Beef, lamb, pork • Seafood with high purine content (eg, sardines, shellfish) • Low-fat or non-fat dairy products • High fructose corn syrup- sweetened sodas, other beverages, or foods • Servings of naturally sweet fruit juices • Table sugar, and sweetened beverages and desserts • Table salt, including in sauces and gravies • Vegetables • Alcohol overuse (defined as more than 2 servings/day for males and 1 serving/day for females) in ALL gout patients • Any alcohol use in gout during periods of frequent gout attacks, or advanced gout with poor control • Alcohol (particularly beer, but also wine and spirits) in all gout patients Figure 4 from Arthritis Care & Research. 2012;64(10):1439.
  • 112. Given the following patient case, identify the options for non-pharmacologic treatment of gout: • RT is 6’4” and 240lbs • He exercises some mornings during the week • RT generally has coffee for breakfast, a pepsi with a roast beef sandwich for lunch, and scallops or steak with vegetables and beer for dinner.
  • 113. • RT is 6’4” and 240lbs • Current Medications: metoprolol XL 50mg po daily, aspirin 81mg po daily, lisinopril 20mg po daily, metformin 1000mg po BID, furosemide 40mg po daily What medication risk factors does RT current have, and can we change any of his medications?
  • 114. • Royal Pains: Season 2 Episode 10 (available on Netflix) – 13:20-14:33 – 28:37-29:45 – 33:25-35
  • 115. • What would be in your differential diagnosis for this patient? • What signs/symptoms of gout does this patient present with? • Is this a typical acute gouty presentation?
  • 117. A.Treatment plan is appropriate B. Treatment plan is inappropriate
  • 119. A. Start the allopurinol as planned. B. Treat the acute attack, then start allopurinol C. Treat the acute attack, then start low dose colchicine and allopurinol concurrently D. Treat the acute attack and modify risk factors
  • 120. • Acute – Non-Pharmacologic: rest, ice – Pharmacologic: • NSAIDs, colchicine, corticosteroids to treat (~7-10 day) • CONTINUE ULT during the acute gouty attack! • Evaluate if indicated for ULT: Tophi, CKD Stage ≥2, Urolithiasis, ≥2 acute gout attacks/year
  • 121. • Chronic – Non-Pharmacologic: diet, fluids, weight loss, smoking cessation, avoidance of risk factors – Pharmacologic: • Prophylactic therapy: LOW DOSE colchicine, NSAIDs, corticosteroids during initiation or just prior to initiation of ULT • Urate lowering therapy (ULT) for indefinite duration dosed to SUA – Allopurinol/febuxostat> probenecid – Probenecid/losartan/fenofibrate may be add on therapy – Pegloticase last line for refractory cases Neoni T. Gout. N Engl J Med. 2011;364:443-52.
  • 123. • We probably wont have time to work on these during class, but Elizabeth Akselrod, PharmD Class of 2015, and I made some practice cases and questions for you to work on at home while studying for the quiz/exam. • I will post our keys to the questions at the end of my version of the ppt slides after our lecture.
  • 124. 1. Which of the following is the goal of urate lowering therapy? A. SUA <6mg/dL B. SUA <4 mg/dL C. SUA <7mg/dL D. Lower the SUA until the tophi resolve 2. Which medication(s) work on xanthine oxidase A. Allopurinol B. Febuxostat C. Pegloticase D. Probenecid E. Both A and B
  • 125. 3) There is new evidence to support dosing allopurinol to a target serum uric acid, instead of renal dose adjustment a) True b) False 3) Low dose colchicine is less effective than high dose colchicine a) True b) False
  • 126. 5) Patients being treated for gout should receive pharmacologic treatment only a) True b) False 6) When initiating urate lowering therapy, which of the following can be given to prevent acute gout attacks a) NSAIDs b) Colchicine c) Corticosteroids d) All of the above
  • 127. • MC is a 21 yoF who presents to her PCP for an annual checkup. Her parents have been concerned because lately she hasn’t been taking good care of herself. She’s been staying up late, eating lots of processed foods, drinking excessive alcohol since her 21st birthday, and has developed a “strange obsession with sticking her tongue out at people.” She presents with no symptoms and simply wants to “get this appointment over with so she can go party in the USA.” • PMH: N/A • SH: Drinks 3-4 mixed drinks per day, quit smoking 4 weeks ago (1 pack per day) • Vitals: 98.6 F, BP 115/76, HR 80, RR 18, O2 Sat 98 • Medications: Women’s MVI, daily laxative use to help with weight loss • Abnormal labs: SUA 7.2 mg/dL • List all MC’s risk factors • Does MC need treatment for gout? If yes, what would you recommend? If no, explain.
  • 128. • RW (prefers to be called Fat Amy) is a 28 yoF who was shooting a film in Hollywood when all of a sudden she felt pain in her right big toe which was red and inflamed. She presented to the ER 3 hours later and stated that her pain was a 4/10. She has no history of acute gouty attacks. • PMH: Type II DM, Obesity, GERD (no ulcers) • SH: 2 beers per night • SUA: 6.5 mg/dL; no tophi present • CrCl: 60 mL/min • What treatment options can you recommend for RW? • Does RW need chronic gout treatment? Prophylaxis? Explain. • Provide one counseling point using patient-specific language.
  • 129. • TS is a 59 yoM who presents to the ER for the 4th time this year with an acute gouty attack. He rates his pain as a 6/10 and says it started 24 hours ago. His history is significant for HTN (refuses to take his Lisinopril because it makes him cough), DMT2, s/p MI in 2009, migraines, gouty attacks with tophi, and s/p vehicle accident with fractured femur. SUA: 8. G6PD (-) and HLA-B*5801 (-). • Recommend a plan for TS. Include drug name, dose, route and duration. • If the patient was HLA-B*5801 (+), which medication would you have to avoid? • Which medication(s) is contraindicated in G6PD deficiency?
  • 130. • SN is a 63 yoM who has no significant PMH except has been taking Allopurinol 300 mg PO daily for 1 year due to frequent gouty attacks starting in 2005. At his annual physical, the doctor notes that SN has chronic knee pain and an SUA of 8.5 mg/dL. The doctor asks for your advice. Since you’re a gout expert, you know exactly what options are available and you recommend: • List 3 non-pharmacologic options to educate SN about:
  • 131. • HR is a 45 yoF who has had breakthrough gouty attacks despite Allopurinol treatment. Her PMH is significant for HTN, DMT2, hyperlipidemia, s/p kidney donation in 2010, and recent start on Azathioprine for RA. She tells the doctor that she has heard of an alternative to Allopurinol that also doesn’t require renal dosing. What is this drug? Is HR eligible to receive this medication? Why/why not?
  • 132. • Dr. Sampson is a new resident and wants to start a patient on Allopurinol for chronic gout treatment. He asks you for help to make sure the patient is eligible to take this medication. What test should be ordered prior to starting the patient on Allopurinol? • Which medications should you beware in the patient’s profile that interacts with Allopurinol?
  • 133. • TY comes into your pharmacy to pick up his newly prescribed probenecid. He has never heard of this medication and knows nothing about it. You check his profile and confirm that he doesn’t have renal impairment and you approve the order. What important points should you make TY aware of?
  • 134. • RC 67 year old African American male comes is a regular patient at your pharmacy. His past medical history includes Hypertension, diabetes, and atrial fibrillation. His current medication list includes: chlorthalidone 25mg po daily, metformin 1000mg po BID, aspirin 81mg po daily, verapamil 80mg po TID, and warfarin 5mg po daily. • VS: within normal limits; BG: 150 (fasting this AM) • He comes to your pharmacy with the following prescription. What would you want to do?
  • 135. Colcrys 1.2mg po once followed by 0.6mg 1 hour later, then 12 hours later start 0.6mg po BID x 7 days. RC 67 7/6/14

Editor's Notes

  1. HANDOUT: In PI there is a table for the potent v moderate and the p-glycoprotein and hepatic renal insufficiency and how to doseFIND THIS AND PUT IT IN THERE Colcrys (colchicine) is a substrate of the efflux transporter PGP. Of the CYP450 enzymes tested, CYP3A4 was mainly involved in the metabolism of colchicine. If colcrys is administered with drugs that inhibit PGP, most of which also inhibit CYP3A4, increased concentrations of colchicine are likely. Fatal drug interactions have been reported.
  2. Colchicine decreases the average number of gout flares, decreases the likelihood of having one or multiple acute flares, and decreases the severity of flares. No change in the length of gout flares. The benefits was observed overall, from 0 to 3 months, and from 3 to 6 months. A significant difference persisted in those subjects continuing colchicine for up to 6 months total Colchicine Placebo P-value Acute Gout Flares 33% of patients 77% of patients 0.008 Multiple Gout Flares 14% of patients 63% of patients 0.004 Visual Analog Scale 3.64 5.08 0.018 Length of Acute Gout Flares 6 days 5.56 days 0.566
  3. We would start ppx as we